Тестове по клинична фармакология за студенти. Предизпитен тест по фармакология

Клиничен фармакологичен тест

система за подготовка на тестове Gee Test oldkyx.com

Списък с въпроси по клинична фармакология

1. Полуживотът на лекарството е:
1) [-] време за достигане на максималната концентрация на лекарството в плазмата;

2) [-] време, през което лекарството достига до системния кръвен поток;

3) [-] времето, през което лекарството се разпределя в тялото;

4) [+] време, през което концентрацията на лекарството в плазмата намалява с 50%;

5) [-] време, през което половината от приложената доза достига до прицелния орган.

2. Ширината на терапевтичното действие е:
1) [-] терапевтична доза от лекарството;

2) [-] съотношението на концентрацията на лекарство в орган или тъкан към концентрацията му в кръвната плазма;

3) [+] диапазонът между минималните терапевтични и минималните токсични концентрации на лекарството в плазмата;

4) [-] процент на лекарството, несвързано с протеина;

5) [-] диапазонът между минималната и максималната терапевтична концентрация на лекарството.

3. Рецепторните агенти на конкурентното действие включват:
1) [-] НСПВС (нестероидни противовъзпалителни средства);

2) [+] β-блокери;

3) [-] бримкови диуретици;

4) [-] нитрати;

5) [-] флуорохинолони.

4. Функцията на черния дроб и бъбреците трябва да се има предвид при предписване на следните лекарства:
1) [-] липофилен, образуващ неактивни метаболити;

2) [+] липофилен, образуващ активни метаболити;

3) [-] хидрофилен;

4) [-] хепатотоксичен;

5) [-] нефротоксичен.

5. Селективността на действието на лекарственото вещество зависи от:
1) [-] полуживот;

2) [-] метод на приложение;

3) [-] връзки с протеини;

4) [-] обем на разпределение;

5) [+] дози.

6. Кинетиката на насищане се характеризира с:
1) [+] увеличаване на полуживота на приложената доза с непроменен клирънс;

2) [-] скоростта на елиминиране е пропорционална на концентрацията на лекарството в плазмата и дозата;

3) [-] полуживотът не е пропорционален на приложената доза.

7. Фактор, определящ необходимостта от преизчисляване на режима на приложение на лекарството при хронична бъбречна недостатъчност:
1) [-] висока липофилност на лекарството;

2) [-] ниска връзка с плазмените протеини;

3) [-] наличието на активни тубулни екскреционни системи;

4) [+] висока степен на екскреция в непроменена форма.

8. Кои лекарства преминават по-лесно през BBB?
1) [-] с висока разтворимост във вода;

2) [+] с висока разтворимост в мазнини;

3) [-] проявяващи свойствата на слаби киселини;

4) [-] проявяващи свойствата на слаби основи;

5) [-] със слабо свързване с плазмените протеини.

9. В какъв случай се получава по-пълно усвояване?
1) [-] абсорбция от стомаха на лекарство, проявяващо свойствата на слаба основа;

2) [-] абсорбция от тънките черва на лекарство, проявяващо свойствата на слаба киселина;

3) [+] абсорбция от тънките черва на лекарство, проявяващо свойствата на слаба основа.

10. Понятието „пресистемен метаболизъм“ включва:
1) [+] биотрансформация на лекарства в черния дроб по време на първото преминаване и в червата;

2) [-] биотрансформация на лекарства в червата;

3) [-] биотрансформация на лекарства в черния дроб по време на първото преминаване и в бъбреците;

4) [-] биотрансформация на лекарства в черния дроб, бъбреците и червата.

11. Следната група странични ефекти е строго зависима от дозата:
1) [-] фармацевтични;

2) [-] фармакогенетичен;

3) [-] алергични;

4) [-] мутагенен;

5) [+] синдром на отнемане.

12. Идентифицирайте група лекарства с тесен терапевтичен индекс:
1) [-] β-блокери;

2) [-] пеницилини;

3) [+] сърдечни гликозиди;

4) [-] АСЕ инхибитори;

5) [-] мощни диуретици.

13. Провеждането на лекарствен мониторинг е желателно при лечение на следната група лекарства:
1) [+] антиконвулсанти;

2) [-] β2-симптомиметици;

3) [-] пеницилини;

4) [-] глюкокортикоиди;

5) [-] М-антихолинергици.

14. Към забавените се отнася следната група нежелани реакции:
1) [-] токсичен;

2) [-] развитие на лекарствена зависимост;

3) [-] фармакогенетичен;

4) [+] канцерогенен;

5) [-] синдром на отнемане.

15. Развитието на асистолия е възможно, когато пропранолол се комбинира с:
1) [-] фенобарбитал;

2) [-] фуроземид;

3) [+] верапамил;

4) [-] фенитоин;

5) [-] ранитидин.

16. Рискът от токсични ефекти се увеличава, когато гентамицин се комбинира с:
1) [+] фуроземид;

2) [-] пеницилин;

3) [-] метилксантини;

4) [-] макролиди;

5) [-] глюкокортикоиди.

17. Рискът от развитие на нежелана бременност се увеличава, когато оралните контрацептиви се комбинират с:
1) [-] хипотония;

2) [-] витамин С;

3) [-] алкохол;

4) [+] тетрациклин;

5) [-] глюкокортикоиди.

18. При бъбречна патология настъпват следните промени във фармакокинетиката на лекарствата, с изключение на:
1) [-] нарушения на бъбречната екскреция;

2) [-] повишаване на концентрацията на лекарства в кръвната плазма;

3) [-] намаляване на свързването с плазмените протеини;

4) [-] увеличение на T1/2;

5) [+] намаляване на бионаличността.

19. Чернодробната цироза се причинява от следните промени във фармакокинетиката на лекарствата, с изключение на:
1) [-] намаляване на метаболизма при първо преминаване;

3) [-] увеличение на Т1/2;

4) [-] повишаване на бионаличността;

5) [+] намаляване на обема на разпределение.

20. При сърдечна недостатъчност се наблюдават следните промени във фармакокинетиката на дигоксин, с изключение на:
1) [-] намаляване на абсорбцията в стомашно-чревния тракт с 30%;

2) [-] намаляване на свързването с плазмените протеини;

3) [+] засилване на метаболизма в черния дроб;

4) [-] намалена бъбречна екскреция;

5) [-] увеличение на T1/2.

21. Алкохолът, когато се приема в големи дози еднократно, води до:
1) [-] повишена абсорбция на лекарството;

3) [+] забавяне на метаболизма в черния дроб;

4) [-] намалена бъбречна екскреция;

5) [-] увеличение на T1/2.

22. Никотинът води до:
1) [-] намалена абсорбция на лекарството;

2) [-] увеличаване на обема на разпространение на наркотици;

3) [-] намалено свързване с плазмения протеин;

4) [+] повишен метаболизъм в черния дроб;

5) [-] повишена бъбречна екскреция на лекарства.

23. Обичайната форма на освобождаване на лекарството се характеризира с:
1) [-] нитронг;

2) [-] сутак-кърлеж;

3) [+] нитросорбид;

4) [-] нифедипин-GITS;

5) [-] верапамил SR.

24. За облекчаване на атака на ангина пекторис се използва сублингвална таблетна лекарствена форма:
1) [-] нитронг;

2) [-] сустак;

3) [+] нитросорбид;

4) [-] атенонолол;

5) [-] верапамил SR.

25. За да се предотврати развитието на толерантност при редовна употреба на нитрати, интервалът без нитрати трябва да бъде:
1) [-] 2-4 часа;

2) [-] 4-6 часа;

3) [-] 6-8 часа;

4) [+] 8-12 часа.

26. За засилване на антиангинозния ефект най-безопасната комбинация е:
1) [-] верапамил + пропранолол;

2) [-] верапамил + атенолол;

3) [-] верапамил + метопролол;

4) [+] верапамил + изосорбид динитрат;

5) [-] верапамил + дилтиазем.

27. Методите за оценка на антиангинозната ефективност на дадено лекарство са всички изброени по-долу, с изключение на:
1) [-] Холтер ЕКГ мониториране;

2) [+] мониториране на дневно кръвно налягане;

3) [-] стрес-ехо;

4) [-] тест на бягаща пътека;

5) [-] VEM проби.

28. При пациент с ангина пекторис в комбинация с артериална хипертония са от полза следните лекарства:
1) [-] нитрати;

29. При ангина при усилие лекарствата по избор са следният клас лекарства:

2) [+] блокери на β-адренергичните рецептори;

3) [-] блокери на α-адренергичните рецептори;

4) [-] имидазолинови рецепторни агонисти;

5) [-] ангиотензин II рецепторни блокери.

30. При вазоспастична стенокардия лекарствата на избор са лекарства от следния клас:
1) [-] блокери на хистаминови рецептори;

2) [-] блокери на β-адренергичните рецептори;

3) [-] блокери на α-адренергичните рецептори;

4) [+] блокери на калциевите канали;

5) [-] ангиотензин II рецепторни блокери.

31. Адекватен метод за проследяване на ефективността и безопасността на антихипертензивната терапия е:
1) [-] ежедневно ЕКГ мониториране;

2) [+] 24-часово проследяване на кръвното налягане;

3) [-] еднократни измервания на кръвното налягане;

4) [-] измерване на показатели за физическа функция;

5) [-] динамика на QT интервала на ЕКГ.

32. Изберете нежелан ефект, който не е типичен за верапамил:
1) [-] брадикардия;

2) [-] запек;

3) [-] развитие на AV блок;

4) [-] подуване на краката и стъпалата;

5) [+] бронхоспазъм.

33. За лечение на артериална хипертония лекарство на първи избор при пациент с хронична сърдечна недостатъчност е:
1) [+] еналаприл;

2) [-] верапамил;

3) [-] клонидин;

4) [-] празозин;

5) [-] нифедипин.

34. Посочете антихипертензивно лекарство, което повишава активността на симпатоадреналната система:
1) [+] нифедипин;

2) [-] клонидин;

3) [-] каптоприл;

4) [-] метопролол;

5) [-] ирбесартан.

35. При артериална хипертония в комбинация със синусова тахикардия се предпочитат:
1) [-] блокери на калциевите канали дихидропиридинови производни;

2) [-] бримкови диуретици;

3) [+] β-блокери;

4) [-] α-блокери;

5) [-] тиазидни диуретици.

36. α1-блокерите са лекарства на избор за лечение на артериална хипертония:
1) [-] при пациенти с чернодробни заболявания;

2) [-] при пациенти с ритъмни нарушения;

3) [+] при възрастни мъже с аденом на простатата и затруднено уриниране;

4) [-] при пациенти с ангина пекторис;

5) [-] при пациенти с анамнеза за инфаркт на миокарда.

37. За лечение на артериална хипертония при пациенти с бронхиална астма не могат да се използват:
1) [-] блокери на калциевите канали;

2) [-] ангиотензин II рецепторни антагонисти;

3) [-] α1-блокери;

4) [+]β-блокери;

5) [-] диуретици.

38. Пациенти с артериална хипертония и анамнеза за инфаркт на миокарда трябва да бъдат предписани на първо място:
1) [+] β-блокери;

2) [-] диуретици;

3) [-] блокери на калциевите канали;

4) [-] имидазолинови рецепторни агонисти;

5) [-] α1-блокери.

39. Лекарства на първи избор при пациенти с артериална хипертония и тежка стеноза на периферните артерии са:
1) [-] неселективни β-блокери;

2) [+] калциеви антагонисти;

3) [-] диуретици;

4) [-] ангиотензин II рецепторни блокери;

5) [-] α2-адренергични рецепторни агонисти.

40. Избройте групите лекарства, които подобряват прогнозата на пациенти с CHF:
1) [-] АСЕ инхибитори;

2) [-] β-блокери;

3) [-] ангиотензин II рецепторни блокери;

4) [-] спиронолактон;

41. Избройте лекарства, които имат директен положителен инотропен ефект:
1) [-] дигоксин;

2) [-] допамин;

3) [-] амринон;

4) [-] левосимендан;

5) [+] всички изброени лекарства.

42. Избройте β-блокери, които имат доказана ефективност при лечението на пациенти с ХСН:
1) [-] атенолол;

2) [-] пропранолол;

3) [+] карведилол;

4) [-] соталол;

43. Посочете показанията за употреба на спиронолактон:
1) [-] ниво на калий в кръвта > 5,5 mmol/l;

2) [-] подуване на краката и стъпалата;

3) [+] сърдечна недостатъчност клас IV по класификацията на NYHA;

4) [-] креатининов клирънс под 30 ml/min;

44. Титрирането на дозата на АСЕ инхибиторите и β-блокерите при пациенти със ЗСН предполага:
1) [-] започване на терапия с минимална доза от лекарството;

2) [-] увеличаване на дозата на лекарството на всеки 2 седмици;

3) [-] постигане на целевата доза на лекарството;

4) [-] намаляване на броя на хоспитализациите и увеличаване на продължителността на живота на пациента;

5) [+] всички изброени знаци.

45. Показания за предписване на амлодипин при ХСН:
1) [-] застойна сърдечна недостатъчност;

2) [+] неконтролирани числа на кръвното налягане;

3) [-] анамнеза за миокарден инфаркт;

4) [-] ритъмни нарушения;

5) [-] всички изброени знаци.

46. ​​​​Принципи на диуретичната терапия за CHF:
1) [-] предписване на диуретици за сърдечна недостатъчност клас II-IV по класификацията на NYHA;

2) [-] загуба на тегло с 0,5-1,0 kg на ден;

3) [-] контрол на кръвното налягане;

4) [-] контрол върху нивото на калий в кръвта;

5) [+] всичко по-горе.

47. Посочете предпочитания начин на приложение на лекарството при застойна сърдечна недостатъчност:
1) [-] сублингвално;

2) [-] ректално;

3) [+] парентерално;

4) [-] устно;

5) [-] всички изброени пътища на администриране.

48. Ефекти на АСЕ инхибитор при пациенти със ЗСН:
1) [-] ефектът на АСЕ инхибитора върху смъртността зависи от продължителността на лечението;

2) [-] намаляването на риска от смърт е по-изразено при пациенти с по-висок FC;

3) [-] наличието на дозозависим ефект на АСЕ инхибитор при пациенти със сърдечна недостатъчност;

4) [+] всичко по-горе.

49. Избройте лекарства, които увеличават продължителността на потенциала на действие:
1) [-] хинидин;

2) [-] прокаинамид;

3) [-] амиодарон;

4) [-] дигоксин;

5) [+] всички изброени лекарства.

50. Лекарства, които удължават QT интервала:
1) [-] клиндамицин;

2) [-] амиодарон;

3) [-] котримоксазол;

4) [-] хинидин;

5) [+] всички изброени лекарства.

51. Лекарства, които удължават PQ интервала:
1) [-] лидокаин;

2) [-] хинидин;

3) [-] дизопирамид;

4) [+] дигоксин;

5) [-] всички изброени лекарства за предписване на антиаритмични лекарства.

52. Показания за предписване на антиаритмични средства са:
1) [-] чести ритъмни нарушения;

2) [-] камерни екстрасистоли - 6 в минута;

3) [-] нарушение на ритъма на високи градации;

4) [+] хемодинамични нарушения;

5) [-] всичко по-горе.

53. Избройте лекарства, които имат антихолинергични странични ефекти:
1) [-] амиодарон;

2) [-] веропамил;

3) [-] лидокаин;

4) [+] хинидин;

5) [-] всички изброени лекарства.

54. Избройте лекарствата, които повишават прага на фибрилация:
1) [-] кордарон;

2) [-] бретилиев тозилат;

3) [-] соталол;

4) [-] пропранолол;

5) [+] всички изброени лекарства.

55. Избройте лекарствата, предписани за предотвратяване на ритъмни нарушения при синдром на WPW:
1) [-] дигоксин;

2) [-] дилтиазем;

3) [+] амиодарон;

4) [-] прокаинамид;

5) [-] всички изброени лекарства.

56. Показания за възстановяване на ритъма при постоянна форма на предсърдно мъждене:
1) [-] чести епизоди на тахисистолия;

2) [-] слабост на синусовия възел;

3) [+] анамнеза за тромбоемболизъм;

4) [-] неефективност на лекарствената терапия;

5) [-] всичко по-горе.

57. Лекарство по избор за лечение на пароксизми на камерна тахикардия:
1) [-] лидокаин;

2) [-] прокаинамид;

3) [-] бретилий;

4) [-] пропафенон;

5) [+] всички изброени лекарства.

58. Странични ефекти на амиодарон:
1) [-] фоточувствителност;

2) [-] дисфункция на щитовидната жлеза;

3) [-] суха кашлица;

4) [-] преходно повишаване на активността на чернодробните аминотрансферази;

5) [+] всички изброени ефекти.

59. Показания за употреба на аденозин:
1) [-] пароксизъм на предсърдно мъждене;

2) [+] реципрочна суправентрикуларна тахикардия;

3) [-] камерна тахикардия;

4) [-] екстрасистол;

5) [-] всичко по-горе.

60. Метод за оценка на ефективността на антиаритмичната терапия в извънболничната практика:
1) [+] Холтер ЕКГ мониториране;

3) [-] тестове с физическа активност;

4) [-] EPI (електрофизиологично изследване);

5) [-] всички изброени методи.

61. Инхалаторните глюкокортикоиди включват:
1) [-] хидрокортизон;

2) [+] беклометазон;

3) [-] преднизолон;

4) [-] полкорталон;

5) [-] дексаметазон.

62. Дългодействащите селективни β2-агонисти включват:
1) [-] флутиказон;

2) [+] салметерол;

3) [-] салбутамол;

4) [-] фенотерол;

5) [-] тербуталин.

63. За облекчаване на пристъп на бронхиална астма се използва:
1) [-] тиотропиев бромид;

2) [-] теопек;

3) [-] натриев кромогликат;

4) [+] салбутамол;

5) [-] будезонид.

64. Дългодействащите антихолинергици включват:
1) [-] ипратропиев бромид;

2) [-] натриев кромикат;

3) [+] тиотропиев бромид;

4) [-] окситропиев бромид;

5) [-] триамцинолон ацетонид.

65. Страничен ефект на инхалаторните глюкокортикостероиди е:
1) [-] главоболие;

2) [-] затлъстяване;

3) [+] орална кандидоза;

4) [-] захарен диабет;

5) [-] полиурия.

66. Муколитичните средства включват:
1) [-] кодеин;

2) [-] натриев кромогликат;

3) [+] ацетилцистеин;

4) [-] салметерол;

5) [-] теофилин.

67. При едновременна употреба повишава концентрацията на теофилин в кръвта:
1) [+] офлоксацин;

2) [-] пеницилин;

3) [-] цефтриаксон;

4) [-] гентамицин;

5) [-] бисептол.

68. Когато се използва едновременно, намалява концентрацията на теофилин в кръвта:
1) [-] пефлоксацин;

2) [-] циметидин;

3) [+] рифампицин;

4) [-] еритромицин;

5) [-] ампиокс.

69. Бронходилататорите не включват:
1) [-] метилксантини;

2) [-] антихолинергици;

3) [-] симпатикомиметици;

4) [+] блокери на левкотриенови рецептори.

70. Основното лечебно средство за хроничен обструктивен бронхит е:
1) [+] тиотропиев бромид;

2) [-] недокромил натрий;

3) [-] фенотерол;

4) [-] монтелукаст;

5) [-] аминофилин.

71. Тахикардията се развива като страничен ефект при приема на всички изброени лекарства, с изключение на:
1) [-] салбутамол;

2) [-] изопротеринол;

3) [-] фенотерол;

4) [-] теофилин;

5) [+] ипратропиев бромид.

72. Комбинираните лекарства за лечение на бронхиална астма не включват:
1) [-] дитек;

2) [-] серетид;

3) [-] симбикорт;

4) [+] беклазон;

5) [-] беродуал.

73. Инхалаторният глюкокортикостероид има най-изразен противовъзпалителен ефект:
1) [-] бекламетазон дипропионат;

2) [-] будезонид;

3) [-] триамцинолон ацетонид;

4) [+] флутиказон пропионат;

5) [-] флунизолид.

74. Лекарство на избор при наличие на инфекции, причинени от метицилин-резистентни щамове на Staphylococcus aureus е:
1) [-] азитромицин;

2) [-] метронидазол;

3) [-] гентамицин;

4) [+] линезолид;

5) [-] цефуроксим.

75. Изберете група антибактериални лекарства за лечение на инфекции, причинени от вътреклетъчни патогени:
1) [+] макролиди;

2) [-] пеницилини;

3) [-] аминогликозиди;

4) [-] цефалоспорини;

5) [-] сулфонамиди.

76. Посочете групата антибактериални лекарства, които имат най-голяма антианаеробна активност:
1) [-] гликопептиди;

2) [-] аминопеницилини;

3) [-] тетрациклини;

4) [-] аминогликозиди;

5) [+] нитроимидазоли.

77. Всички изброени антибактериални лекарства са нефротоксични, с изключение на:
1) [-] гентамицин;

2) [-] карбеницилин;

3) [+] азитромицин;

4) [-] цефазолин;

5) [-] ванкомицин.

78. Посочете антибактериално лекарство, което има слаба активност срещу пневмококи:
1) [-] азитромицин;

2) [-] пеницилин;

3) [-] цефтриаксон;

4) [+] ципрофлоксацин;

5) [-] хлорамфеникол.

79. Изберете рационална комбинация от антибактериални лекарства, която има синергично действие срещу грам-положителни микроорганизми и безопасност:
1) [-] пеницилини + тетрациклини;

2) [-] пеницилини + цефалоспорини;

3) [-] аминогликозиди + гликопептиди;

4) [+] пеницилини + аминогликозиди;

5) [-] пеницилини + сулфонамиди.

80. Следните антибактериални лекарства проникват добре през кръвно-мозъчната бариера:
1) [-] линкозамиди;

2) [-] макролиди;

3) [-] тетрациклини;

4) [-] аминогликозиди;

5) [+] III поколение цефалоспорини.

81. Лекарството на избор при лобарна пневмония е:
1) [-] ципрофлоксацин;

2) [-] доксициклин;

3) [-] гентамицин;

4) [-] цефотаксим;

5) [+] бензилпеницилин.

82. Лекарство на избор при тонзилофарингит е:
1) [+] цефуроксим аксетил;

2) [-] доксициклин;

3) [-] цефтазидим;

4) [-] офлоксацин;

5) [-] фурагин.

83. Лекарствата на избор при инфекции на жлъчните пътища са:
1) [-] аминогликозиди;

2) [-] нитрофурани;

3) [+] III поколение цефалоспорини;

4) [-] макролиди;

5) [-] естествени пеницилини.

84. При болнични инфекции, възникващи в интензивни отделения, лекарствата по избор са следните комбинации от антибактериални лекарства:
1) [-] ампицилин + гентамицин;

2) [+] цефтазидим + амикацин;

3) [-] цефуроксим + еритромицин;

4) [-] клиндамицин + гентамицин;

5) [-] норфлоксацин + пеницилин.

85. Предпочитаната група антибактериални лекарства за лечение на хроничен простатит са:
1) [-] сулфонамиди;

2) [-] карбапенеми;

3) [-] хинолони;

4) [-] линкозамини;

5) [+] флуорохинолони.

86. Изберете лекарство, което максимално потиска секрецията на солна киселина:
1) [-] пирензепин;

2) [-] циметидин;

3) [-] мизопростол;

4) [-] антиациди;

5) [+] омепразол.

87. Максималният брой странични ефекти сред H2 блокерите има:
1) [+] циметидин;

2) [-] роксатидин;

3) [-] низатидин;

4) [-]ранитидин;

5) [-] фамотидин.

88. Инхибира цитохром P-450:
1) [-] омепразол;

2) [-] пирензепин;

3) [+] циметидин;

4) [-] фамотидин;

5) [-] лансопразол.

89. Синдромът на откат се причинява от:
1) [-] синтетични простагландини;

2) [-] антиациди;

4) [-] М-антихолинергици;

5) [+] H2 блокери.

90. При язва на дванадесетопръстника е рационално да се предписват антиациди:
1) [-] преди хранене;

2) [-] по време на хранене;

3) [+] 1,5-2 часа след хранене;

4) [-] 5 часа след хранене;

5) [-] независимо от приема на храна.

91. За профилактика на язви, причинени от приема на НСПВС, най-ефективни са следните:
1) [-] антиациди;

2) [-] H2 блокери;

3) [-] блокери на протонната помпа;

4) [+] синтетични простагландини;

5) [-] М-антихолинергици.

92. При ерадикационната терапия се използва следният антибиотик за унищожаване на H. pylori:
1) [-] карбеницилин;

2) [-] еритромицин;

3) [-] цефоперазон;

4) [+] кларитромицин;

5) [-]хлорамфеникол.

93. Развитието на резистентност на H. pylori към:
1) [-] ванкомицин;

2) [+] метронидазол;

3) [-] тетрациклин;

4) [-] нитрофурани;

5) [-] цефотаксим.

94. Бактерициден ефект срещу H. pylori има:
1) [-] сукралфат (Вентър);

2) [+] бисмутов субцитрат (де-нол);

3) [-] алмагел;

4) [-] фамотидин;

5) [-] пирензепин.

95. При ерадикационната терапия за унищожаване на H. pylori се използват:
1) [-] антиациди;

2) [-] синтетични простагландини;

3) [+] блокери на "протонната помпа";

4) [-] М-антихолинергици;

5) [-] гликопептиди.

96. Кортикостероидите повишават токсичността:
1) [-] теофилин;

2) [+] тиазидни диуретици;

3) [-] златни препарати;

4) [-] противоязвени лекарства.

97. Ефектът на глюкокортикоидите намалява:
1) [-] циметидин;

2) [-] аспирин;

3) [-] диклофенак;

4) [-] амиодарон;

5) [+] рифампицин.

98. Най-висока минералкортикоидна активност има:
1) [-] полкартолон;

2) [-] преднизолон;

3) [+] хидрокортизон;

4) [-] дексаметазон.

99. След прием на НСПВС ефектът се развива по-бързо:
1) [-] противовъзпалително;

2) [+] аналгетик;

3) [-] антикоагулант.

100. Най-силно изразено аналгетично свойство има:
1) [-] ацетилсалицилова киселина;

2) [-] ибупрофен;

3) [-] напроксен;

4) [+] парацетамол.

101. Механизмът за развитие на улцирогенния ефект на НСПВС е:
1) [-] повишена киселинност на стомашния сок;

2) [+] намален синтез на простагландини в стомашната лигавица;

3) [-] намалено възстановяване на лигавицата.

102. Най-силно изразените противовъзпалителни свойства са:
1) [+] фенилбутазон;

2) [-] метамизол;

3) [-] пироксикам;

4) [-] парацетамол;

5) [-] ибупрофен.

103. Ранен страничен ефект на глюкокортикоидите е:
1) [-] катаракта;

2) [-] миопатия;

3) [-] остеопороза;

4) [-] Кушингоиден синдром;

5) [+] стероиден диабет.

104. Посочете ефект, който не е характерен за кортикостероидите:
1) [-] противовъзпалително;

2) [-] антиалергично;

3) [-] анти-шок;

4) [-] имуносупресивен;

Преглед:

Предмет: "Фармацевтични лекарства, повлияващи периферната нервна система"

Тестови задачи

1.Адреналинът причинява:

Изберете един отговор.

а.) Намалена консумация на кислород

Б.) Хипергликемия

В.) Инхибиране на гликогенолизата

D.) Инхибиране на липолизата

2.Адреналинът е противопоказан при:

Изберете един отговор.

а.) Тиреотоксикоза

Б.) Анафилактичен шок

В.) Сърдечен блок

D.) Хипогликемична кома

3. Блокер на ганглии:

Изберете един отговор.

а.) атропин;

B.) пипекуроний;

В.) пентамин;

D.) сукцинилхолин (дитилин).

4.Ганглийните блокери се използват за лечение на:

Изберете един отговор.

а.) запек.

Б.) хипертонична криза;

В.) глаукома;

Г.) задържане на урина;

5. Действието на M-ChR агониста е блокирано:

Изберете един отговор.

а.) Цитизин

Б.) Тубокурарин

В.) Прозерин

D.) Атропин

Д.) Пилокарпин

6. За спиране на действието на конкурентните мускулни релаксанти се използва следното:

Изберете един отговор.

а.) атропин;

Б.) дипироксим.

В.) неостигмин (прозерин);

7. Селективен М-холиномиметик (агонист на мускариновите холинергични рецептори):

Изберете един отговор.

а.) Прозерин

Б.) Пилокарпин

В.) Цитизин

D.) Физостигмин

E.) Карбахолин

8. Адсорбиращите агенти включват:

Изберете един отговор.

а.) Нишестена слуз.

Б.) Отвара от дъбови кори;

В.) Танин;

Г.) Активен въглен;

9. Дразнещите вещества включват всичко с изключение на:

Изберете един отговор.

а.) Основен бисмутов нитрат;

Б.) Ментол.

В.) Пречистено терпентиново масло (терпентин);

D.) Хартия с горчица;

10.M-антихолинергични:

Изберете един отговор.

а.) пентамин;

B.) пипекуроний;

В.) сукцинилхолин (дитилин).

D.) атропин;

11.M-антихолинергиците причиняват развитието на мидриаза:

Изберете един отговор.

а.) повишаване на тонуса на радиалния мускул на ириса;

Б.) намаляване на тонуса на орбикуларния ирисов мускул;

В.) повишаване на тонуса на цилиарния мускул.

12.M-антихолинергиците се използват за лечение на:

Изберете един отговор.

а.) артериална хипертония;

Б.) глаукома;

В.) стомашна язва.

Г.) миастения;

13.М-антихолинергиците са противопоказани при:

Изберете един отговор.

а.) бронхиална астма;

Б.) глаукома;

В.) атриовентрикуларен блок;

D.) стомашна язва.

14.М-холиномиметиците, за разлика от инхибиторите на ChE, не засягат холинергичната синаптична трансмисия:

Изберете един отговор.

а.) на нервно-мускулната връзка

B.) от постганглионарните аксони на автономните нерви до ефектора (гладки мускули, екзокринни жлези)

В.) в централната нервна система

15. Местните анестетици се използват в комбинация с адреналин, защото:

Изберете един отговор.

а.) абсорбцията на анестетика се ускорява и локалният анестетичен ефект се засилва.

Б.) забавя се абсорбцията на анестетика и се засилва локалният анестетичен ефект;

В.) абсорбцията на анестетика се забавя и локалният анестетичен ефект отслабва;

16. Метопролол е показан за лечение на:

Изберете един отговор.

а.) Атриовентрикуларен блок

Б.) Бронхиална астма

В.) Доброкачествена хиперплазия на простатата

Г.) Артериална хипертония

17. Механизмът на действие на адстрингентите се дължи на:

Изберете един отговор.

а.) блок на натриевите канали;

Б.) адсорбция на химични съединения;

В.) покриване на лигавиците с филм, който предотвратява дразненето на сетивните нерви. г.) ​​коагулация на протеини и образуване на филм, който предпазва окончанията на сетивните нерви от дразнене;

18. Механизмът на действие на локалните анестетици се дължи на:

Изберете един отговор.

а.) Блокиране на калциевите канали и удължаване на абсолютния рефрактерен период;

Б.) Блокиране на калиеви канали и невъзможност за реполяризация на мембраната;

В.) Активиране на хлоридни канали и хиперполяризация.

Г.) Блок на натриевите канали и невъзможност за деполяризация на мембраната;

19. Мускулен релаксант:

Изберете един отговор.

а.) скополамин.

B.) пипекуроний;

В.) атропин;

D.) пентамин;

20. Нежелан страничен ефект на ганглиоблокери:

Изберете един отговор.

а.) хипертонична криза;

Б.) повишено вътреочно налягане.

В.) ортостатичен колапс;

Г.) бронхоспазъм;

21. Неселективен бета-блокер:

Изберете един отговор.

а.) Метопролол

Б.) Атенолол

В.) Празозин

D.) Пропранолол

22. Неостигмин (прозерин) се използва за лечение на миастения гравис, тъй като подобрява холинергичното синаптично предаване:

Изберете един отговор.

а.) В автономния ганглий

B.) На мионевралната връзка

C.) От постганглионарни холинергични влакна до клетки на ефекторни органи

23. Норепинефринът се повишава:

Изберете един отговор.

а.) Периферно съдово съпротивление

Б.) Мотилитет на стомашно-чревния тракт

В.) Бронхиален тонус

D.) Сърдечна честота

24. Защо атропинът (третичен амин) превъзхожда метацина (кватернерно амониево съединение) в действието си върху централната нервна система:

Изберете един отговор.

а.) по-добре разпределени в тялото (> Vd стойности);

Б.) се абсорбира по-добре в системното кръвообращение от мястото на инжектиране (> коефициент на бионаличност);

C.) се екскретира (елиминира) от тялото по-бавно (> стойности на T1/2).

25. Защо галантаминът (третичен амин) превъзхожда прозерина (кватернерно амониево съединение) в действието си върху централната нервна система:

Изберете един отговор.

а.) По-бавно елиминиране от тялото (> T1/2 стойности)

B.) По-добре се абсорбира в системното кръвообращение от мястото на инжектиране (> коефициент на бионаличност)

C.) По-добро разпределение в тялото (>Vd стойности)

26. Празозинът причинява:

Изберете един отговор.

а.) Намален тонус на гладката мускулатура на бронхите

Б.) Намалено вътреочно налягане

В.) Намаляване и отслабване на сърдечните контракции

D.) Намаляване на периферното съдово съпротивление

27. Пропранололът причинява:

Изберете един отговор.

а.) Намален стомашно-чревен мотилитет

B.) Контракция на мускула на ириса (мидриаза)

В.) Намален бронхиален тонус

D.) Намален пулс

28. Рефлексната брадикардия се причинява от:

Изберете един отговор.

а.) Салбутамол

B.) Празозин

В.) Метопролол

D.) Карведилол

Д.) Норепинефрин

29. Съкратителната активност на миометриума се намалява от:

Изберете един отговор.

а.) Салбутамол

Б.) Карведилол

В.) Пропранолол

D.) Норепинефрин

Д.) Метопролол

30. Лекарства за лечение на остра съдова недостатъчност:

Изберете един отговор.

а.) Метопролол

Б.) Салбутамол

В.) Добутамин

D.) Норепинефрин

E.) Пропранолол

31. Само за повърхностна анестезия се използва:

Изберете един отговор.

а.) Бензокаин (анестетик).

B.) Бупивакаин;

В.) прокаин (новокаин);

D.) Лидокаин;

32. Холиномиметиците са противопоказани при:

Изберете един отговор.

а.) Миастения гравис

Б.) Болест на Алцхаймер

В.) Бронхиална астма

D.) Ксеростомия

Д.) Глаукома

Преглед:

Предмет: "Химиотерапевтични средства"

Тестови задачи

1. Механизмът на действие на антигените върху протеиновия синтез в микробните клетки се основава на способността им да инхибират:

Изберете един отговор.

а.) ДНК полимераза

Б.) РНК полимераза

В.) процес на транспептидация

D.) процесът на четене на кода на m-RNA

2. Изберете антибиотик - инхибитор на протеиновия синтез в бактериална клетка:

Изберете един отговор.

а.) бензилпеницилин

Б.) карбеницилин

В.) ампицилин

D.) гентамицин

3. Изберете дефиницията на „химиотерапия“

Изберете един отговор.

а.) химиотерапията е потискане на патогени на повърхността на човешкото тяло (кожа, лигавици)

Б) Химиотерапията е потискането на патогени в околната среда (предмети за грижа, инструменти, секрети на пациента)

В.) химиотерапията е въздействие върху клетките на макроорганизма

D.) химиотерапията е потискане на патогени във вътрешната среда на макроорганизма

4. Принципите на химиотерапията включват следното:

Изберете един отговор.

а.) всички отговори са верни

Б.) лечението с антибактериални лекарства трябва да започне възможно най-скоро след началото на заболяването

В.) лекарството трябва да бъде избрано, като се вземе предвид чувствителността на патогена към химиотерапевтичния агент;

Г.) дозата на химиотерапията трябва да се предписва, като се вземе предвид тежестта на заболяването на пациента

5. Нитрофурановите производни включват:

Изберете един отговор.

а.) Фталилсулфатиазол (фталазол)

Б.) налидиксова киселина

В.) фуразолидон

D.) нитрохексолин

6. Кои от следните антибиотици нарушават синтеза на клетъчната стена:

Изберете един отговор.

а.) хлорамфеникол

Б.) тетрациклини

В.) бета-лактамни антибиотици

D.) полимиксини

7. Кои лекарства за лечение на онихомикоза дават най-ниска честота на рецидиви?

Изберете един отговор.

а.) тербинафин и итраконазол;

Б.) амфотерицин В и нистатин;

В.) гризеофулвин и леворин;

D.) цинков ундецилинат и йод

8. Какви лекарства са ефективни срещу респираторни синцитиални вируси и грипни вируси?

Изберете един отговор.

а.) рибавирин, интерферон;

Б.) озелтамивир, ремантадин

C.) азидотимидин, саквинавир;

D.) ацикловир, фамцикловир;

9. Кое от твърденията правилно отразява един от общите принципи на химиотерапията при инфекциозни заболявания.

Изберете един отговор.

Б.) клиничното подобрение е основание за прекъсване на терапията

В.) ефективността на лечението често не зависи от продължителността на антибиотичната терапия. д. 3) след клинично подобрение лечението не трябва да се спира и да продължи още 48-72 часа, ако е необходимо.

10. Кой антибиотик е ефективен при псевдомембранозен колит?

Изберете един отговор.

а.) диклоксациклин

Б.) фуразолидон

В.) ванкомицин

D.) ампицилин

11. Кой от следните антибиотици е бета-лактам:

Изберете един отговор.

а.) стрептомицин

Б.) меропенем

В.) тетрациклин

D.) полимиксин

12. Кое от химиотерапевтичните лекарства принадлежи към сулфонамидите:

Изберете един отговор.

а.) менкомицин

Б.) еритромицин

В.) стрептомицин

D.) сулфадимин

13. Кой макролид има най-нисък клирънс?

Изберете един отговор.

а.) еритромицин

B.) азитромицин

В.) кларитромицин

D.) рокситромицин

14. Кое перорално лекарство е ефективно при гъбичен менингит (напр. криптококов)?

Изберете един отговор.

а.) амфотерицин В;

Б.) флуконазол

C.) тербинафин;

D.) кетоконазол;

15. Кой бензилпеницилинов препарат се класифицира като биосинтетичен:

Изберете един отговор.

а) ампицилин

B.) бензилпеницилин-бензатин

В.) азлоцилин

D.) карбеницилин

16. Какво лекарство се използва за лечение на стомашно-чревна кандидоза?

Изберете един отговор.

а) клотримазол

Б.) гризеофулвин;

В.) нитрофунгин;

D.) нистатин;

17. Какво лекарство се използва за лечение на системни микози?

Изберете един отговор.

а.) нистатин;

Б.) клотримазол

В.) амфотерицин В;

D.) гризеофулвин;

18. Кое лекарство е ефективно срещу грипни вируси тип А и В?

Изберете един отговор.

а) ремантадин

Б.) ацикловир;

C.) азидотимидин;

D.) озелтамивир;

19. Механизмът на действие на сулфонамидите е свързан с:

Изберете един отговор.

а) инхибиране на COX

Б.) инхибиране на дихидрофолат редуктазата;

C.) конкурентен антагонизъм с PABA и инхибиране на дихидроптероат синтетазата

D.) конкурентен антагонизъм с GABA

20. Най-честото усложнение на бета-лактамните антибиотици е

Изберете един отговор.

а.) аритмии

Б.) инхибиране на хемопоезата

В. 1) алергични реакции

D.) загуба на слуха

21. Полимиксините се считат за лекарства от 3-та линия („дълбок резерв“), защото:

Изберете един отговор.

а.) имат ниска ефективност

Б.) широко разпространена съпротива срещу тях

В.) поради ниска антимикробна активност

D.) поради висока органотоксичност

22. При използване на резорбтивни сулфонамиди са възможни следните нежелани реакции:

Изберете един отговор.

а.) агранулоцитоза

Б.) всичко по-горе

В.) кристалурия

Г.) хемолитична анемия, метхемоглобинемия

23. Антивирусните лекарства (AVS) са най-ефективни, когато лечението започне рано, защото:

Изберете един отговор.

а.) PVA показва визуален ефект;

B.) PVA проявява вируциден ефект;

C.) PVA не проявява органотоксичност

D.) PVA е органотоксичен;

24. Посочете антиретровирусни лекарства (за лечение на HIV инфекция):

Изберете един отговор.

а.) арбидол, озелтамивир;

B.) азидотимидин, саквинавир;

В.) ацикловир, фамцикловир;

D.) интерферон, ганцикловир

25. Посочете механизма на действие на флуорохинолоните:

Изберете един отговор.

а.) увеличаване на пропускливостта на CPM

B.) Инхибиране на синтеза на бактериална стена

C.) инхибиране на PDEase

D.) инхибиране на ДНК гираза

26. Посочете лекарство, свързано с оксазолидиноните:

Изберете един отговор.

а.) линезолид

B.) моксифлоксацин

В.) ко-тримоксазол

D.) линкомицин

27. Посочете антихерпетично средство:

Изберете един отговор.

а.) азидотимидин;

Б.) ацикловир;

В.) арбидол;

D.) саквинавир

28. Какво е характерно за доксициклина?

Изберете един отговор.

а.) лошо се абсорбира от стомашно-чревния тракт

Б.) бионаличността намалява, когато се приема с храна

C.) T1/2 16-24 часа

D.) основният път на елиминиране е през MVP

Преглед:

Предмет : "Обща фармакология"

Тестови задачи

1 . Веществата с афинитет и вътрешна активност се наричат:

Изберете един отговор.

а.) антагонисти

B.) агонисти

2 . Действието на веществото, което се развива след навлизането му в системното кръвообращение, се нарича:

Изберете един отговор.

а.) резорбтивна

Б.) местен

В.) страничен продукт

Г.) рефлекс

3 . Как се нарича действието на веществото, ако то взаимодейства само с функционално недвусмислени рецептори с определена локализация и не засяга други рецептори?

Изберете един отговор.

а) рефлекс

Б.) обратимо

В.) необратимо

Г.) селективен

4 . Как се нарича натрупването на лекарствени вещества в организма при многократно приложение?

Изберете един отговор.

а.) тахифилаксия

Б.) материална кумулация

В.) идиосинкразия

D.) сенсибилизация

5 . Как се нарича намаляването на ефективността на дадено вещество, когато то се повтаря?

Изберете един отговор.

а.) толерантност (пристрастяване)

Б.) кумулация

В.) идиосинкразия

Г.) пристрастяване

6. Как се нарича явлението, когато спирането на наркотици причинява психични и соматични разстройства, свързани с дисфункция на много системи на тялото, дори смърт?

Изберете един отговор.

а.) синдром на отнемане

Б.) оттегляне

В.) сенсибилизация

Г.) идиосинкразия

7. Кой от процесите протича по време на фазата на биотрансформация, която се нарича конюгация?

Изберете един отговор.

а.) хидролиза

Б.) възстановяване

В.) подкисляване

D.) ацетилиране

8. Кой отговор отговаря най-добре на термина „рецептор”?

Изберете един отговор.

а.) активни групи от субстратни макромолекули, с които лекарството взаимодейства

B.) транспортни системи, активирани от лекарства

C.) активирани от лекарства редокс ензими

D.) йонни канали на биологични мембрани, чиято пропускливост се променя от лекарственото вещество

9. Кой фармакокинетичен параметър е обозначен като “T1/2”:

Изберете един отговор.

а.) константа на скоростта на елиминиране

Б.) полуживот (полуживот, полуелиминиране) на веществата

В.) абсорбция от мястото на инжектиране на 50% от веществото

D.) общ просвет

10. Метаболитната биотрансформация е:

Изберете един отговор.

а.) взаимодействие с глюкуронова киселина

Б.) превръщане на вещество в резултат на окисление, редукция, хидролиза

В.) свързване с албумин в кръвната плазма

Г.) метилиране и ацетилиране на вещества

11. Обемът на разпространение на лекарствата отразява:

Изберете един отговор.

а.) съотношението на еднократната и дневната доза на лекарственото вещество

B.) хипотетичен обем на течността, в която се разпределя лекарството

В.) изчисленото количество лекарство, което достига до системното кръвообращение

D.) съотношение доза-телесно тегло

12. Обемът на разпределение е нисък, ако:

Изберете един отговор.

а.) веществото се намира в плазмата, интерстициалната и вътреклетъчната течност и се натрупва в тъканите

B.) веществото се намира в плазмата и интерстициалната течност

В.) веществото се намира в плазмата, интерстициалната и вътреклетъчната течност

D.) веществото се натрупва в кръвната плазма

13. Отбележете основния механизъм на усвояване на лекарствата:

Изберете един отговор.

а.) пиноцитоза

Б.) пасивна дифузия

В.) активен транспорт

D.) филтриране

14. Фармакокинетиката включва:

Изберете един отговор.

а.) биотрансформация на лекарствени вещества в организма

Б.) Ефект на лекарствата върху генетичния апарат

В.) усложнения на лекарствената терапия

Г.) Ефектът на лекарствата върху метаболизма в организма

15.Какво включва понятието фармакодинамика?

Изберете един отговор.

а.) метаболизъм на лекарствата в организма

Б.) условия за съхранение на лекарствата

В.) биологични ефекти на лекарствата

D.) метод на приложение на лекарството

16. Какво включва понятието „Биотрансформация”:

Изберете един отговор.

а.) свързване на вещества с протеините на кръвната плазма

Б.) натрупване на вещества в мастната тъкан

В.) комплекс от физикохимични и биохимични трансформации на лекарствено вещество, насочени към отстраняването му от тялото

D.) натрупване на лекарството в мускулната тъкан

17.Как се нарича вътрешната активност на веществото?

Изберете един отговор.

а.) способността на вещество, когато взаимодейства с рецептор, да го разпознае

Б.) способността на веществото да взаимодейства с транспортни системи

В.) способността на веществото, когато взаимодейства с рецептор, да го стимулира и да предизвика биологичен ефект

D.) способността на веществото да взаимодейства с плазмените протеини

18. Какво означава терминът „афинитет“?

Изберете един отговор.

а.) афинитетът на веществото към транспортните системи на тялото

Б.) афинитет на веществото към албумин в кръвната плазма

В.) афинитет на лекарства към микрозомални чернодробни ензими

Г.) афинитетът на вещество към рецептор, водещ до образуването на комплекс „вещество-рецептор“ с него

19. Какво означава терминът "бионаличност":

Изберете един отговор.

а.) степен на свързване на веществата с плазмените протеини

Б.) количеството вещество в урината спрямо началната доза на лекарството

В.) способност за преминаване през кръвно-мозъчната бариера

D.) количеството непроменено вещество, достигнало до кръвната плазма, спрямо началната доза на лекарството

20. Какво отговаря на понятието „активен транспорт“:

Изберете един отговор.

а.) инвагинация на клетъчната мембрана с образуване на вакуола

Б.) транспорт срещу концентрационен градиент с консумация на енергия

C.) транспорт по концентрационен градиент без консумация на енергия

D. 1) улеснена дифузия

Преглед:

Предмет : „Имунотропни средства“

Тестови задачи

1. Блокерите на H1-хистаминовите рецептори се използват за всички изброени по-долу показания ОСВЕН:

Изберете един отговор.

а.) уртикария;

Б.) бронхиална астма

В.) лекарствени алергии;

Г.) сезонен ринит;

2.Какви видове фармакодинамични ефекти на глюкокортикоидите се използват в медицината?

Изберете един отговор.

а.) всички са правилни с изключение на 1

Б.) хипергликемия, потискане на растежните зони на епифизите;

В.) противошокови, детоксикиращи (индукция на чернодробни ензими);

Г.) всичко по-горе;

Д.) имуносупресивно, антиалергично, противовъзпалително;

3. Кои лекарства са най-ефективни като противовъзпалителни средства?

Изберете един отговор.

а.) широкоспектърни антибиотици

B.) НСПВС;

В.) SPVA;

D.) стабилизатори на мембраната на мастоцитите;

4. Какви са приемливите критерии за ефективност на инсулиновата терапия?

Изберете един отговор.

а.) еугликемия, еуглюкозурия;

Б.) еугликемия, аглюкозурия;

В.) агликемия, аглюкозурия

Г.) нормогликемия, еуглюкозурия;

5. Какви са локалните нежелани реакции при системно използване на GCS-съдържащи мехлеми и кремове?

Изберете един отговор.

а.) подуване, хиперемия, болка;

Б.) остеопороза, хирзутизъм, дисменорея

В.) хипертрофия, хиперпигментация, кандидоза;

Г.) повишен риск от локални инфекции, атрофия, депигментация;

6. Кои са най-опасните нежелани реакции на системните кортикостероиди при продължителна употреба?

Изберете един отговор.

а) синдром на отнемане (надбъбречна недостатъчност);

B.) всичко по-горе;

В.) Синдром на Иценко-Кушинг („Кушингоид“);

Г.) 1 и 2 са верни.

Д.) имунодефицитно състояние;

7. Какви са показанията за употребата на гестагенни лекарства?

Изберете един отговор.

а.) хормонозаместителна терапия след овариохистеректомия;

Б.) рак на гърдата, рак на простатата;

В.) дисфункционално маточно кървене, повтарящ се спонтанен аборт, ендометриоза, контрацепция;

D.) контрацепция при жени с висока хипердислипидемия, захарен диабет и анамнеза за холестаза

8. Кое е най-честото усложнение при лечение с инсулинови лекарства?

Изберете един отговор.

а.) липодистрофия;

Б.) хипокалиемия;

В.) инсулинова резистентност

Г.) хипогликемия;

9. Кой GCS препарат има ниска бионаличност при локално приложение (върху кожата)?

Изберете един отговор.

а.) будезонид;

Б.) флуоцинолон ацетонид (синафлан);

В.) преднизолонов хемисукцинат

D.) бекламетазон пропионат;

10. Кое лекарство от инхалаторните кортикостероиди осигурява най-малък риск от системни нежелани реакции?

Изберете един отговор.

а.) преднизолон хемисукцинат

Б.) бекламетазон пропионат;

C.) будезонид;

D.) флуоцинолон ацетонид (синафлан);

11. Кое лекарство е инсулинов сенсибилизатор?

Изберете един отговор.

а.) метформин;

Б.) хумулин

C.) пиоглитазон;

D.) акарбоза;

E.) глибенкламид;

12. Кое лекарство се използва при сезонни алергични реакции (сенна хрема) само като превантивно средство?

Изберете един отговор.

а.) клемастин;

Б.) хидрокортизон;

C.) натриев кромогликат;

Г.) всичко по-горе

13. H1-хистаминовите блокери от второ поколение се различават от лекарствата от първо поколение

Изберете един отговор.

а.) изразен седативен ефект;

Б.) антиеметичен ефект

В.) значителен М-антихолинергичен ефект;

Г.) по-голяма селективност на действие;

14. Окситоцинът се характеризира с всички свойства ОСВЕН

Изберете един отговор.

а.) чувствителността на матката е постоянно висока

Б.) ефективен в малки дози като родостимулатор;

В.) ефективен в големи дози като утеротоник;

Г.) чувствителността на матката към него се повишава по време на раждането;

15. Показанията за употреба на имуносупресивни цитостатици включват всичко ОСВЕН:

Изберете един отговор.

а.) автоимунни заболявания;

Б.) предотвратяване на ГНИЛЕНЕ

В.) тежки анафилактични реакции;

16. При хипотиреоидизъм се използва като средство за заместителна терапия

Изберете един отговор.

а.) протирелин;

Б.) калиев йодид;

В.) тиреотропин

D.) левотироксин;

17. Използването на цитостатични имуносупресори (метотрексат, флуороурацил, циклофосфамид) често е сложно

Изберете един отговор.

а.) левкопения и инфекциозен синдром;

Б.) алергии и фотодерматити;

В.) кървене и анемия;

D.) сънливост и летаргия

18. Тиамазол (мерказолил) като основно (дългосрочно) лечение е показан за...

Изберете един отговор.

а.) рак на щитовидната жлеза;

Б.) микседем

В.) нодуларна токсична гуша;

Г.) дифузна токсична гуша;

19. Посочете правилната последователност на употребата на лекарства за анафилактичен шок:

Изберете един отговор.

а.) преднизолон – клемастин – аминофилин – епинефрин;

Б.) клемастин (тавегил) - епинефрин (адреналин) - преднизолон - аминофилин (аминофилин)

в.) епинефрин – преднизолон – клемастин – аминофилин

20. Как да спрем диабетната кома?

Изберете един отговор.

а.) IV 40-80 ml 40% разтвор на глюкоза;

B.) IV 1 ml 0,1% разтвор на адреналин

В.) интравенозно 20 единици инсулин-цинкова суспензия;

Г.) интравенозно краткодействащ инсулин 0,1 единици/час;

21. Какво НЕ е абсолютно противопоказание за употребата на естрогенни лекарства?

Изберете един отговор.

а.) маточно кървене с неизвестен характер;

Б.) чернодробно заболяване, анамнеза за жълтеница;

Д.) тромбофилия;

Преглед:

Предмет : „Лекарства, повлияващи

относно функцията на изпълнителните органи"

Тестови задачи

1. Лекарства за спешно лечение на хипертонична криза (когато се появят или засилят признаци на увреждане на таргетните органи):

Изберете един отговор.

а.) метилдопа;

Б.) каптоприл;

В.) натриев нитропрусид

D.) метопролол;

2.ААС за лечение на суправентрикуларни и камерни тахиаритмии:

Изберете един отговор.

а.) Верапамил

Б.) Лидокаин

C.) Прокаинамид (новокаинамид)

D.) Фенитоин (дифенин)

3.ААС с минимален потенциал за проаритмогенно действие:

Изберете един отговор.

а.) Пропранолол (Анаприлин)

Б.) Амиодарон

В.) Пропафенон

D.) Лидокаин

4. AAS, използвани за лечение на коронарна артериална болест:

Изберете един отговор.

а.) Лидокаин

Б.) Верапамил

В.) Хинидин

D.) Пропафенон

5.AAS, характеризиращ се с най-дълъг полуживот:

Изберете един отговор.

а.) Хинидин

Б.) Аденозин

В.) Лидокаин

D.) Амиодарон

6. Антихипертензивен невротропен агент с периферно действие:

Изберете един отговор.

а.) каптоприл;

Б.) метопролол;

В.) нифедипин

D.) клонидин;

7. Антихипертензивно лекарство от групата на блокерите на калциевите канали:

Изберете един отговор.

а.) нифедипин

Б.) метопролол;

В.) каптоприл;

D.) лозартан;

8. Антихипертензивно лекарство от групата на миотропните вазодилататори:

Изберете един отговор.

а.) каптоприл;

Б.) дилтиазем;

C.) дихлоротиазид;

D.) метопролол

9. Антихипертензивно средство с централно действие:

Изберете един отговор.

а.) клонидин;

Б.) пентамин

В.) натриев нитропрусид;

D.) каптоприл;

10. Антихипертензивно средство, което блокира алфа и бета адренергичните рецептори: .

Изберете един отговор.

а.) карведилол;

Б.) метопролол

C.) атенолол;

D.) пропранолол;

11. Антихипертензивно лекарство, за което има висок риск от развитие на ефекта от първата доза (тежка хипотония в ортостатично положение):

Изберете един отговор.

а.) метопролол;

B.) хидрохлоротиазид;

В.) каптоприл;

D.) празозин

12. Антихипертензивно лекарство, противопоказано при двустранна стеноза на бъбречната артерия:

Изберете един отговор.

а.) метопролол;

Б.) верапамил;

В.) каптоприл;

D.) нифедипин

13. Антихипертензивно средство, което намалява образуването на ангиотензин II:

Изберете един отговор.

а.) верапамил;

B.) лосартан;

В.) каптоприл;

D.) празозин

14. Антихипертензивно средство, което намалява нивото на ренин в кръвта:

Изберете един отговор.

а.) празозин;

Б.) верапамил;

В.) пропранолол

D.) пентамин;

15. Има антифибринолитичен ефект:

Изберете един отговор.

а.) фитоменадион

Б.) калциев хлорид

В.) хепарин

D.) аминокапронова киселина

16. Бета-блокерите се използват при исхемична болест на сърцето въз основа на:

Изберете един отговор.

а.) намаляване на нуждата от кислород чрез намаляване на сърдечната честота и контрактилитета на миокарда; b.) намаляване на нуждата от кислород на миокарда чрез намаляване на предварителното натоварване;

В.) повишено извличане на O2 от кръвта

Г.) подобряване на коронарния кръвоток;

17. Вещество, което активира фибринолизата е:

Изберете един отговор.

а.) Варфарин

Б.) клопидогрел

В.) хирудин

D.) стрептокиназа

18. Всички кардиотоници повишават:

Изберете един отговор.

а.) атриовентрикуларна проводимост;

Б.) контрактилитет на миокарда;

В.) автоматизъм на синоатриалния възел

Г.) потребност от кислород на миокарда;

19. Основният елемент на антихипертензивното действие на алфа-блокерите:

Изберете един отговор.

а.) Венозна вазодилатация;

B.) Отрицателни хроно- и инотропни ефекти

В.) Артериоларна вазодилатация;

20. Основният елемент на антихипертензивното действие на бета-блокерите:

Изберете един отговор.

а.) Отрицателни хроно- и инотропни ефекти

B.) Артериоларна вазодилатация;

В.) Венозна вазодилатация;

D.) Блокада на рецепторите за ангиотензин II;

21. За лечение на артериална хипертония НЕ се прилагат:

Изберете един отговор.

а.) фуроземид

B.) спиронолактон;

C.) хидрохлоротиазид;

D.) манитол;

22. За системно лечение на артериална хипертония НЕ се прилагат:

Изберете един отговор.

а.) метопролол;

B.) лосартан;

В.) фентоламин

D.) нифедипин;

23. Бронходилататорите от групата на симпатикомиметиците включват:

Изберете един отговор.

а.) изадрин

Б.) ефедрин

В.) салбутамол

24. Негликозидните кардиотоници включват всички лекарства ОСВЕН на:

Изберете един отговор.

а.) милринон

Б.) строфантин (уабаина);

В.) допамин;

D.) добутамин;

25. Индиректните антикоагуланти включват:

Изберете един отговор.

а.) хирудин

B.) Натриев хидроген цитрат

В.) фраксипарин

D.) варфарин

26. Сърдечните гликозиди (CG) включват всички изброени по-долу лекарства, С ИЗКЛЮЧЕНИЕ на:

Изберете един отговор.

а.) дигоксин;

B.) добутамин;

В.) дигитоксин;

D.) строфантин

27. Коя комбинация от диуретици е рационална?

Изберете един отговор.

а.) Фуроземид + манитол

B.) Манитол + урея

В.) Дихлоротиазид + триамтерен

D.) Фуроземид + етакринова киселина

28. Какви нитроглицеринови препарати използват пациентите с ангина за облекчаване на пристъпите?

Изберете един отговор.

а.) нитроглицерин в мехлем

Б.) нитроглицерин в сублингвални таблетки;

В.) нитроглицерин в микродраги (сустак);

D.) нитроглицерин в разтвор за венозно приложение;

29. Какви признаци на предозиране на SG са животозастрашаващи?

Изберете един отговор.

а.) умора, мускулна слабост

Б.) диспептични разстройства;

В.) зрителни смущения;

Г.) камерни тахиаритмии;

30.Какви лекарства се използват за лечение на исхемична болест на сърцето?

Изберете един отговор.

а.) всичко по-горе

Б.) антиатеросклеротични средства;

В.) антитромботични средства;

D.) кардиопротективни средства;

31. Какво лекарство може да облекчи чревните спазми (колики)?

Изберете един отговор.

а.) метамизол (аналгин);

B.) метоклопрамид;

В.) дротаверин (но-шпа).

D.) морфин;

E.) магнезиев сулфат;

32. Кой антиацид може да причини алкалоза при системна употреба?

Изберете един отговор.

а.) магнезиев трисиликат;

Б.) алуминиев хидроксид;

В.) калциев глюконат;

D.) натриев бикарбонат

E.) магнезиев оксид;

33. Какъв антиеметик се прилага при рефлукс и стомашна пареза?

Изберете един отговор.

а.) ондансетрон (зофран);

Б.) хлорпромазин (аминазин);

C.) метоклопрамид;

D.) дифенхидрамин (дифенхидрамин);

E.) перфеназин хидрохлорид (етаперазин)

34. Кой диуретик може да причини загуба на слуха?

Изберете един отговор.

а.) спиронолактон

B.) Дихлоротиазид

В.) манитол

D.) фуроземид

35. Какво лекарство се използва за подобряване на контрактилната активност на миометриума по време на раждане?

Изберете един отговор.

а.) ергометринов малеат

B.) Атропин сулфат

В.) окситоцин

D.) папаверин

36. Какво лекарство се използва за спиране на кървене от матката?:

Изберете един отговор.

а.) ергометринов малеат

B.) Атропин сулфат

В.) фенотерол

D.) простагландин F-2a

37. Кое лекарство е антикоагулант с директно действие?

Изберете един отговор.

а.) фибринолизин

Б.) фитоменадион

В.) Хепарин

D.) Варфарин

38. Кое лекарство се използва само за предотвратяване на повръщане, причинено от болест на пътуването (морска болест)?

Изберете един отговор.

а.) метоклопрамид (церукал);

B.) перфеназин хидрохлорид (етаперазин)

В.) дипразин (пиполфен);

D.) "Aeron";

E.) ондансетрон (Zofran);

39. Кое лекарство намалява контрактилитета на миометриума?

Изберете един отговор.

а.) фенотерол

Б.) питуитрин

C.) простагландин F-2a

D.) Папаверин

40. Механизмът на бронходилататорното действие на атропина е свързан с:

Изберете един отговор.

а.) директен миотропен ефект върху гладката мускулатура на бронхите

B.) блокада на m-холинергичните рецептори на бронхиалната гладка мускулатура

В.) стимулиране на В2-адренорецепторите

41. Механизъм на действие на бримковите диуретици (фуроземид и др.):

Изберете един отговор.

а.) Увеличете осмотичното налягане на течността в лумена на тубулите

Б.) Намаляване на реабсорбцията на натрий, хлорид и калий в дебелата част на възходящия крайник на бримката на Хенли

в.) Повишаване на гломерулната филтрация

D.) Блокирайте карбанхидразата

42. Механизъм на действие на тиазидните диуретици?

Изберете един отговор.

а.) Увеличете осмотичното налягане на течността в тубулите на нефрона

B.) Увеличете скоростта на гломерулна филтрация

C.) Блокирайте алдостероновите рецептори

г.) ​​Намаляване на реабсорбцията на натрий и хлор в дисталните тубули

43. Механизмът на отхрачващо действие на препаратите от термопсис се дължи на:

Изберете един отговор.

а) дразнене на стомашните рецептори и рефлексно засилване на секрецията на бронхиалните жлези

б.) директно стимулиране на секрецията на бронхиалните жлези

в.) втечняване на храчки по време на деполимеризация на протеини

44. Най-подходящото показание за предписване на СГ е:

Изберете един отговор.

а.) нестабилна стенокардия;

б.) ХСН с тежка брадикардия;

в.) ХСН с множество камерни екстрасистоли

г.) ​​ХСН с предсърдно мъждене;

45. Общо свойство на всички AAS (с изключение на сърдечните гликозиди), използвани за лечение на тахиаритмии:

Изберете един отговор.

а.) Забавяне на бързата деполяризация

б.) Забавяне на реполяризацията

в.) Ускоряване на реполяризацията

г.) ​​Намален автоматизъм

46. ​​Основните свойства на хепарина са:

Изберете един отговор.

а.) Кумулира

б.) ефективен, когато се приема през устата

в.) Действието се развива след 18-24 часа

d.) Забавя съсирването на кръвта "in vivo" и "in vitro"

47. Характеристики на действието на лидокаин:

Изберете един отговор.

а.) Забавя бързата деполяризация

б.) Ускорява реполяризацията

в.) Забавя проводимостта

г.) ​​Повишава кръвното налягане

48. Обърнете внимание на антитромбоцитния агент - COX инхибитор:

Изберете един отговор.

а.) варфарин

б.) фитоменадион

в.) ацетилсалицилова киселина

г.) ​​Натриев хидроген цитрат

49. Обърнете внимание на пряко действащия коагулант:

Изберете един отговор.

а.) фитоменадион

б) тромбин

в.) апротинин

г.) ​​Хепарин

50. Обърнете внимание на страничните ефекти на еуфилин:

Изберете един отговор.

а.) респираторна депресия

б.) повишена нужда от кислород на миокарда

в.) повишено кръвно налягане

51. Маркирайте лекарството, свързано с хидроуретиците:

Изберете един отговор.

а. 2) индапамид

b. 3) манитол

° С. 1) Дихлоротиазид

д. 4) фуроземид

52. Маркирайте лекарството, свързано със салуретиците:

Изберете един отговор.

а.) Урея

б.) Манитол

в.) Демеклоциклин

г.) ​​Фуроземид

53. Маркирайте лекарството, което подобрява изтичането на жлъчка (холекинетик):

Изберете един отговор.

а.) “Холензим”;

б.) дехидрохолева киселина;

в.) дротаверин (но-шпа);

г.) ​​магнезиев сулфат;

д.) атропин;

е.) аминофилин (аминофилин)

54. Обърнете внимание на холесекретното лекарство от растителен произход:

Изберете един отговор.

а.) Магнезиев сулфат;

б.) осалмид (оксафенамид);

в.) “Алохол”;

г.) ​​"Холензим"

д.) дротаверин (но-шпа);

55. Маркирайте лаксатив за спешно прочистване на червата (подготовка за терапевтични или диагностични процедури):

Изберете един отговор.

а.) лактулоза;

б.) магнезиев сулфат;

в.) изафенин;

г.) ​​глицеринови супозитории;

д.) фенолфталеин

56. Избройте средствата за заместителна терапия при хроничен панкреатит:

Изберете един отговор.

а.) контрикал;

б.) пентагастрин

c.) мизопростол;

г.) ​​атропин;

д.) панкреатин;

57. Избройте лекарство, което намалява секрецията на солна киселина в стомаха:

Изберете един отговор.

а.) омепразол;

б.) натриев бикарбонат;

в.) алуминиев хидроксид

г.) ​​хистамин;

д.) пентагастрин

58. Защо коронарните литични лекарства (например дипиридамол) могат да причинят "феномена на кражба" на миокарда?

Изберете един отговор.

а.) тонизира коронарните съдове;

б.) повишават контрактилитета на миокарда

в.) преразпределят кръвния поток към здрави съдове в ущърб на исхемичната област на миокарда;

г.) ​​разширяват съдовете на системното кръвообращение;

59. При бронхиална астма се използва лекарство, което намалява бронхиалната реактивност от групата на GC:

Изберете един отговор.

а.) бекламетазон дипропионат

б.) кромолин натрий

в.) ипратропиев бромид

60. Лекарството по избор за облекчаване на бронхоспазъм е:

Изберете един отговор.

а.) изадрин

б) салбутамол

в.) атропин

61. При какви заболявания се използват диуретици за рутинна терапия:

Изберете един отговор.

а.) остро отравяне

б.) Мозъчен оток

в.) хипертония

г.) ​​Белодробен оток

62. При белодробен оток се използва следното за намаляване на налягането в белодробната циркулация:

Изберете един отговор.

а.) блокери на ганглии

б.) вдишване на кислород

в.) дихателни стимуланти

63. При белодробен оток вдишването на разтвор на етилов алкохол се използва за:

Изберете един отговор.

а.) противопенно действие

б.) наркотичен ефект

в.) дехидратиращ ефект

64. Антиаритмичният ефект на сърдечните гликозиди се дължи на:

Изберете един отговор.

а.) Намалена сила на сърдечните контракции

б.) Забавяне на проводимостта

в.) Намален автоматизъм

г.) ​​Намалена възбудимост

65. Средство против кашлица, което потиска кашличния рефлекс и блокира възбудимостта на чувствителните окончания в дихателните пътища е:

Изберете един отговор.

а.) тусупрекс

б) кодеин

в.) либексин

66. Следното има смесен стимулиращ ефект върху дихателния център:

Изберете един отговор.

а.) кофеин

б.) никетамид (кордиамин)

в.) cititon

67. Специфичен нежелан страничен ефект на инхибитори на ангиотензин-конвертиращия ензим:

Изберете един отговор.

а.) суха кашлица

б.) агранулоцитоза;

в.) ринорея;

г.) ​​анорексия;

68.Лекарство за лечение на брадиаритмии

Изберете един отговор.

а.) Верапамил

в.) Лидокаин

г.) ​​Атропин

69.Лекарство само за лечение на камерни тахиаритмии

Изберете един отговор.

а.) Пропафенон

б.) Прокаинамид (новокаинамид)

в.) Лидокаин

г.) ​​Верапамил

70.Лекарство само за лечение на суправентрикуларни тахиаритмии

Изберете един отговор.

а.) Лидокаин

б.) Прокаинамид (новокаинамид)

в.) Верапамил

г.) ​​Пропафенон

71. Посочете най-честата нежелана реакция на нитратите:

Изберете един отговор.

а.) образуване на метхемоглобин;

б.) главоболие;

в.) понижен тонус на жлъчния мехур

г.) ​​потискане на тромбоцитната агрегация;

72. Какво е показанието за метоклопрамид?

Изберете един отговор.

а.) диария;

б.) ниска киселинност;

в.) повишена киселинност;

г.) ​​кинетози (морска, въздушна болест);

д.) гадене, повръщане.

Преглед:

Предмет:„Лекарства, повлияващи

върху централната нервна система"

Тестови задачи

1. Какво е вярно за ASC?

Изберете един отговор.

а.) не се използва при артрит;

б.) най-малко улцерогенен;

в.) да не се използва при деца под 14 години с температура;

г.) ​​като антитромбоцитно средство, използвано в дози, по-големи от аналгетичните

2. Какво НЕ е противопоказание за употребата на опиоидни аналгетици?

Изберете един отговор.

а.)респираторна депресия;

б.) миокарден инфаркт;

в.) черепно-мозъчна травма

г.) ​​остра коремна болка с неясен произход;

3. Какво се използва за възстановяване на дишането при предозиране на хероин (морфин)?

Изберете един отговор.

а.)налоксон;

б.) кислород;

c.)трамадол;

г.) ​​налтрексон

4. Какво е характерно за антипиретичния ефект на ненаркотичните аналгетици?

Изберете един отговор.

а.) НА причиняват хипотермия чрез потискане на производството на топлина;

б.) назначаването е задължително при субфебрилитет;

c.) НА намаляват температурата чрез увеличаване на топлообмена

г.) ​​присъщ е на всички НА в дози значително по-високи от аналгетичните;

5. Какво е характерно за пиразолоновите производни (метамизол (аналгин), фенилбутазон (бутадион))?

Изберете един отговор.

а.) използвани като антиагреганти за исхемична болест на сърцето

б.) използва се за продължителна терапия на артрит;

в.) хематотоксичен;

г.) ​​няма противовъзпалителен ефект;

6. Какво е характерно за противовъзпалителния ефект на НСПВС?

Изберете един отговор.

а.) подобряване на качеството на живот на пациентите, временно намаляване на симптомите на артрит;

б.) излекуване на артрит с пълен курс на лечение;

в.) инхибират всички фази на възпалението;

г.) ​​противовъзпалителният ефект се дължи на инхибирането на синтеза на левкотриени7. Каква е причината за смъртта при предозиране на опиоидни аналгетици?

Изберете един отговор.

а.) бронхоспазъм;

б.) белодробен оток;

в.) спиране на дишането;

г.) ​​сърдечен арест

8. Екстрапирамидни двигателни нарушения - типичен нежелан страничен ефект:

Изберете един отговор.

а.) Клозапин

б.) Халоперидол

в.) Оланзапин

г.) ​​Рисперидон

9.Какво лекарство се използва при епилептичен статус?

Изберете един отговор.

а.) дифенхидрамин (дифенхидрамин);

б.)диазепам;

в.) етосуксимид

10. Какъв ефект на опиоидните аналгетици може да бъде опасен дори при еднократна употреба на фона на силна болка

Изберете един отговор.

а.) спазми в стомашно-чревния тракт;

б.)еуфория;

в) запек

г.)респираторна депресия;

11. Какъв ефект на опиоидните аналгетици ограничава широкото им използване?

Изберете един отговор.

а.) аналгетик;

б.)успокоително;

° С. 3) еуфоричен;

д. 4) спазмогенен

12. Кофеин:

Изберете един отговор.

а.) Тонизира дихателния и вазомоторния център

б.) Свива коронарните съдове

в.) Разширява мозъчните съдове

13. Моклобемид, в сравнение с имипрамин, има по-силен:

Изберете един отговор.

а.) Психостимулиращ ефект

б.) Психоседативен ефект

в.) Алфа адренергичен блокиращ ефект

г.) ​​М-антихолинергичен ефект

14. Нимезулид и целекоксиб - селективни COX-2 инхибитори - се различават от неселективните (ASA, диклофенак и др.):

Изберете един отговор.

а.) по-голяма ефективност;

б.) по-ниска честота на гастропатия;

в.) по-малко алергенни;

г.) ​​значително по-ниска тежест на всички „PG-зависими” странични ефекти

15. Основно показание за употреба на опиоидни (наркотични) аналгетици

Изберете един отговор.

а) травматична и висцерална болка с висок интензитет

б.) травматична и висцерална болка с умерен интензитет;

в.) невралгия;

г.) ​​остеоалгия;

16. Обърнете внимание на нежеланите реакции, общи за NA/NSAIDs („COX- и PG-зависими“):

Изберете един отговор.

а.) пристрастяване, наркотична зависимост;

б.) летаргия, респираторна депресия;

в.) алергични реакции, левкопения

г.) ​​гастропатия, кървене;

17. Проверете правилното твърдение за кеторолак:

Изберете един отговор.

а.) ефективен само при умерена болка;

б.) използва се за продължителна терапия на артрит;

в.) използван за не повече от 5-7 дни поради нефротоксичност

г.) ​​хепатотоксичен;

18. При паркинсонизъм се използва:

Изберете един отговор.

а.)фенитоин (дифенин);

b.)карбамазепин;

в.)леводопа

19. Антиеметичен ефект има:

Изберете един отговор.

а.) Хлорпромазин

б.) Буспирон

в.) Зопиклон

г.) ​​Диазепам

20. Антиконвулсивен ефект има:

Изберете един отговор.

а.) Халоперидол

б.) Диазепам

в.) Буспирон

г.) ​​Хлорпромазин

21. Психостимулант от групата на метилксантините:

Изберете един отговор.

а.) Амфетамин

б.) Кофеин

в.) Моклобемид

г.) ​​Пирацетам

д.) Имипрамин

22. Специфично лечение при остро отравяне с бензодиазепинови производни:

Изберете един отговор.

а.) Зопиклон

б.) Флумазенил

в.) Кофеин

г.) ​​Пирацетам

д.) Феназепам

23. Транквилизатор с най-дълъг полуживот (T1/2 >48 часа):

Изберете един отговор.

а.) Диазепам

б.) Оксазепам

в.) Лоразепам

г.) ​​Медазепам

д.) Мидазолам

24. Трицикличен антидепресант:

Изберете един отговор.

а.) Кофеин

б.) Амитриптилин

в.) Флуоксетин

г.) ​​Пирацетам

25. Посочете характеристиките на аналгетичния ефект на NA:

Изберете един отговор.

а.) ефективен при умерена артралгия, миалгия, цефалгия;

б.) премахване на болка от всякакъв интензитет;

в.) по-ефективен от наркотичните аналгетици при силна травматична и висцерална болка;

г.) ​​при продължителна употреба се развива толерантност

26. Флуоксетин в сравнение с амитриптилин::

Изберете един отговор.

а.) По-малко токсичен

б.) Има по-силен М-антихолинергичен ефект

c.) Характеризира се с по-голяма клинична ефективност

г.) ​​Има по-силен седативен ефект

27. Как се различават хипнотичните производни на барбитуровата киселина от бензодиазепиновите производни?

Изберете един отговор.

а.) изразен централен миорелаксиращ ефект

б.) повече нарушения на структурата на съня;

в.) слаба индукция на микрозомални чернодробни ензими;

28. Как се различават частичните агонисти и агонистите-антагонисти на опиоидните рецептори (пентазоцин, бупренорфин) от пълните агонисти (морфин)?

Изберете един отговор.

а.) по-силен спазмогенен ефект;

б.) по-малка наркогенност;

c.) възможно ректално приложение

г.) ​​по-голяма респираторна депресия;

29. Алостеричен активатор на GABA-A рецептори:

Изберете един отговор.

а.) Баклофен

б.) Диазепам

в.) Буспирон

г.) ​​Амизил

30. Антидепресант селективен MAO-A инхибитор:

Изберете един отговор.

а.) Моклобемид

б.) Пирацетам

в.) Флуоксетин

г.) ​​Имипрамин

д.) Амитриптилин

е.) Кофеин

31. Антидепресант, селективен инхибитор на обратното захващане на серотонина:

Изберете един отговор.

а.) Пирацетам

б.) Флуоксетин

в.) Кофеин

г.) ​​Имипрамин

д.) Амитриптилин

32. Няма антиманиакален ефект:

Изберете един отговор.

а.) Халоперидол

б.) Литиев карбонат

в.) Диазепам

г.) ​​Трифтазин

33. Антипсихотично лекарство от групата на фенотиазиновите производни:

Изберете един отговор.

а.) Рисперидон

б.) Оланзапин

в.) Хлорпромазин

г.) ​​Клозапин

д.) Халоперидол

34. Атипичен антипсихотик:

Изберете един отговор.

а.) Флуорофеназин

б.) Халоперидол

в.) Клозапин

г.) ​​Хлорпромазин

д.) Трифтазин

35. Основният психотропен ефект на пирацетам:

Изберете един отговор.

а.) Анксиолитик

б.) Успокоително

в.) Мнемотропен

г.) ​​Психостимулант

36. НСПВС се характеризират с всички изброени по-долу лекарствени взаимодействия ОСВЕН:

Изберете един отговор.

а.) кодеинът отслабва аналгетичния ефект на NA или НСПВС;

б.) НСПВС отслабват ефекта на диуретиците и някои антихипертензивни лекарства;

в.) алуминий-съдържащите антиациди намаляват бионаличността на НСПВС

г.) ​​седативите усилват аналгетичния ефект на НСПВС;

37. Ден успокоително:

Изберете един отговор.

а.) Феназепам

б.) Зопиклон

в.) Медазепам

г.) ​​Диазепам

д.) Аминазин

38. Камерни тахиаритмии потенциален нежелан страничен ефект:

Изберете един отговор.

а.) Типични антипсихотици

б.) Трициклични антидепресанти

в.) Антидепресанти, селективни инхибитори на обратното захващане на серотонина

г.) ​​Транквиланти на бензодиазепиновите производни

д.) Атипични антипсихотици

39. Симптомите на остро отравяне със сънотворни включват:

Изберете един отговор.

а.) възбуда, повишено кръвно налягане;

б.) кома, респираторна депресия, хипоксия;

в.) повишена температура, повишена рефлекторна възбудимост

40. Кои опиоидни аналгетици са противопоказани при миокарден инфаркт?

Изберете един отговор.

а.) пентазоцин, буторфанол;

б) морфин, промедол;

в.) фентанил, налбуфин

41.Какви лекарства могат да се използват при спастичност на скелетните мускули?

Изберете един отговор.

а.) стрихнин, никетамид (кордиамин), бемегрид

б) баклофен, диазепам, мидокалм;

в.) прозерин, галантамин, физостигмин;

42. Кое твърдение относно ацетаминофен (парацетамол) НЕ е вярно?

Изберете един отговор.

а.) НСПВС по избор за артрит

б.) гастротоксични;

в.) няма антитромбоцитен ефект;

г.) ​​антипиретик на избор при вирусни инфекции при деца;

43. Кой високоефективен опиоиден аналгетик е за предпочитане за облекчаване на болката по време на краткотрайни болезнени процедури/операции?

Изберете един отговор.

а.) морфин;

б) пентазоцин

c.) фентанил;

г.) ​​промедол;

44. Кое лекарство се класифицира като антиепилептично лекарство?

Изберете един отговор.

а.)натриев валпроат;

б.)леводопа;

в.) циклодол

45. Кое лекарство е хипнотик?

Изберете един отговор.

а.)циклодол;

б.) зопиклон;

c.)фенитоин (дифенин);

г.)леводопа

46. ​​Кое лекарство е за предпочитане за обезболяване в първия етап на раждането?

Изберете един отговор.

а) кодеин

б.) метамизол (аналгин);

c.) морфин;

d.) тримеперидин (Promedol);

а.) медни препарати

б.) фосфор

в.) живачни съединения

г.) ​​железни съединения

2. За кое лекарствено вещество е основната фармакологична реакция поради ефекта върху процесите на транскрипция на ДНК:

Изберете един отговор.

а.) инсулин;

б.) бензилпеницилин

в.) хепарин;

г.) ​​преднизолон;

3. За кое лекарствено вещество основната фармакологична реакция е причинена от намаляване на пропускливостта на волтаж-зависимите йонни канали:

Изберете един отговор.

а.) дигитоксин;

б.) лидокаин;

в.) ропин;

г.) ​​фуроземид

4. За кое лекарствено вещество е основната фармакологична реакция, причинена от намаляване на пропускливостта на медиатор-зависимите (хемочувствителни) йонни канали:

Изберете един отговор.

а.) лидокаин;

б) пипекуроний

в.) парацетамол;

г.) ​​верапамил;

5. За кое лекарствено вещество е основната фармакологична реакция поради инхибиране на ензимната активност:

Изберете един отговор.

а.) лидокаин;

б.) адреналин;

в.) прозерин

г.) ​​атропин;

6. За кое лекарствено вещество е първичната фармакологична реакция поради инхибиране на процеса на улеснена дифузия:

Изберете един отговор.

а.) адреналин;

б.) дихлоротиазид.

c.) дигоксин;

г.) ​​диазепам;

7. За да намалите концентрацията на отрова в кръвта и тъканите, използвайте:

Изберете един отговор.

а.) лаксативи

б.) химически противоотрови

в.) адсорбенти

г.) ​​функционални антидоти

8. За да се отстрани неабсорбираната отрова от стомаха, последният се измива с вода с добавяне на:

Изберете един отговор.

а.) разтвор на атропин

б.) натриев сулфат

в.) метилтионинов хлорид (метиленово синьо)

г.) ​​активен въглен

9. Комплексоните включват:

Изберете един отговор.

а.) пентацин

б.) налоксон

в.) натриев тиосулфат

г.) ​​пентамин

10.Какви средства се използват за стимулиране на дихателния център:

Изберете един отговор.

а.) никетамид (кордиамин); бемегрид; сулфокамфокаин;

б.) морфин; фентанил; тримеперидин (промедол)

в.) епинефрин (адреналин); фенилефрин (мезатон); норепинефрин (норепинефрин)

г.) ​​дротоверин (но-шпа); метацин; папаверин;

11. Натриевият тиосулфат образува нискотоксични тиоцианатни съединения в случай на отравяне:

Изберете един отговор.

а.) хероин

б) цианиди

в.) атропин

г.) ​​сърдечни гликозиди

12. Основните цели на лечението на остро отравяне са всички, с изключение на:

Изберете един отговор.

а.) намаляване на концентрацията на отрова в кръвта и тъканите

б.) намаляване на по-нататъшното усвояване на отровата

в.) нормализиране на функциите на жизненоважни органи и системи

г.) ​​забавяне на метаболизма на отровата

13. Функционалният антидот на морфина е:

Изберете един отговор.

а.) дифенхидрамин (дифенхидрамин)

б) атропин

в.) налоксон

г.) ​​бемережа

14. Химическият антидот при предозиране с хепарин е:

Изберете един отговор.

а.) фитоменадион

б.) калциев хлорид

в.) протамин сулфат

г.) ​​димеркапрол (унитиол)

15. Етиловият алкохол променя метаболизма на отровата по време на отравяне:

Изберете един отговор.

а.) метилов алкохол

б) атропин

в.) морфин

г.) ​​арсенови препарати


1.Какво е името на клона на фармакологията, който изучава абсорбцията, разпределението, биотрансформацията и екскрецията на лекарства?

Фармакокинетика.

Фармакодинамика.

2.Какво е името на клона на фармакологията, който изучава видовете действие на лекарствата, фармакологичните ефекти и механизма на действие?

Фармакодинамика.

Фармакокинетика.

3. Основният механизъм на абсорбция на лекарството в стомашно-чревния тракт:

Активен транспорт.

Улеснена дифузия.

Пасивна дифузия през клетъчните мембрани.

Пиноцитоза.

4. Основното място на абсорбция на лекарството е слабите основи:

Тънко черво.

5. Основното място на абсорбция на лекарството са слабите киселини:

Тънко черво.

6. Кой метод на приложение на лекарството осигурява 100% бионаличност?

Интрамускулно.

Ректално.

Интравенозно.

През устата.

7. Как ще се промени усвояването на лекарствата - слаби киселини - при намаляване на киселинността на стомашния сок?

Ще нарастне.

Ще намалее.

8. Как ще се промени абсорбцията на лекарства - слаби основи - при намаляване на киселинността на стомашния сок?

Ще нарастне.

Ще намалее.

9. Веществата се транспортират лесно чрез пасивна дифузия през биологични мембрани:

Липофилни.

Полярен.

Хидрофилен.

10. Ентерален начин на приложение на лекарството:

Интрамускулно.

Вдишване.

Под езика.

Интравенозно.

11. Парентерален начин на приложение на лекарството:

През устата.

В ректума.

Подкожно.

Под езика.

12. Къде се извършва абсорбцията на повечето лекарства?

В устната кухина.

В стомаха.

В тънките черва.

В дебелото черво.

13. Следното може да се прилага интравенозно:

Маслени разтвори.

Неразтворими съединения.

Осмотично активни съединения.

Микрокристални суспензии.

Неразтворими съединения.

14.Какво функционално изменение в организма предизвикват сърдечните гликозиди при сърдечна недостатъчност?

Възбуда.

Потисничество.

Тонизиране.

Спокоен.

15.Каква функционална промяна в организма причинява лекарство, което понижава кръвното налягане при артериална хипертония?

Възбуда.

Потисничество.

Тонизиране.

Спокоен.

16.Как се нарича натрупването на лекарство в организма при многократно приложение?

Функционална кумулация.

Сенсибилизация.

Материална кумулация.

Тахифилаксия.

17. Толерантността е:

Алергична реакция на тялото към многократно приложение на лекарството.

Намаляване на фармакологичния ефект при многократно приложение на лекарството.

Непреодолимо желание отново да вземете лекарства.

18. Намаляването на ефекта при прилагане на лекарства на кратки интервали е:

Тахифилаксия.

Идиосинкразия.

Сенсибилизация.

Пристрастяване.

19. Страничен ефект, който може да възникне самопри многократно приложение на лекарства:

Идиосинкразия.

Тератогенен ефект.

Мутагенен ефект.

Пристрастяване.

20. Страничен ефект, който може да възникне самопри употреба на психотропни лекарства:

Идиосинкразия.

Пристрастяване.

Пристрастяване.

Сенсибилизация.

21. Определете вида на лекарственото взаимодействие: пациент с отравяне с мускарин е подложен на стомашна промивка със суспензия от активен въглен:

Обобщена синергия.

Химически антагонизъм.

Конкурентен антагонизъм.

Физически антагонизъм.

22. Мутагенният ефект е:

23. Тератогенният ефект е:

Увреждане на генетичния апарат на зародишната клетка.

Нарушена диференциация на тъканите на плода, причинявайки различни аномалии.

Страничен ефект, който се проявява през първите 12 седмици след оплождането и причинява смъртта на ембриона.

24. Ембриотоксичен ефект е:

Увреждане на генетичния апарат на зародишната клетка.

Нарушена диференциация на тъканите на плода, причинявайки различни аномалии.

Страничен ефект, който се проявява през първите 12 седмици след оплождането и причинява смъртта на ембриона.

КЛИНИЧНА ФАРМАКОЛОГИЯ

001. Как се променя обемът на разпределение на мастноразтворимите лекарства?

при пациенти със затлъстяване?

а) намалява

в) не се променя

г) не се променя или се увеличава

г) се увеличава

002. Как се променя биотрансформацията на лекарствата поради тютюнопушене и пиене на алкохол?

а) намалява

б) намалява или не се променя

в) не се променя

г) не се променя или се увеличава

г) засилва се

003. Кой е основният фактор, който определя биоеквивалентността на лекарството?

а) фармакодинамични характеристики на лекарството

б) физични и химични характеристики

в) лекарствена форма

г) технология на производство

д) състоянието на тялото на пациента

004. Какви странични ефекти на лекарствата не зависят от дозата?

а) свързани с фармакологичните свойства на лекарствата

б) токсични усложнения,

причинени от абсолютно или относително предозиране

в) вторични ефекти поради нарушение

имунобиологични свойства на организма

г) имунологични реакции от незабавен и забавен тип

д) синдром на отнемане

005. Какво усложнение при новородени

може ли прилагането на магнезиев сулфат на бременни жени преди раждането да причини?

а) развитие на невромускулни блокади и летаргия

б) респираторна депресия

в) тромбоцитопения

г) хепатотоксичен ефект

д) недохранване

006. Какви усложнения имат новородените?

Може ли предписването на адренергични блокери за бременни жени да причини това?

а) загуба на слуха

б) увреждане на кожата

в) хипотрофия на плацентата и плода

г) преждевременно затваряне на ductus botallus

д) хеморагичен синдром

007. Приложение на кои антимикробни лекарства

най-безопасен по време на бременност?

а) аминогликозиди

б) котримоксазол

в) нитрофурани

г) пеницилини

д) флуорохинолони

008. Посочете антибиотик, който няма антипсевдомонадна активност:

а) карбеницилин

б) ампиокс

в) тикарцилин

д) азлоцилин

г) цефтазидим

009. Пациент с пневмония на антибактериално лечение,

започна да се оплаква от световъртеж,

нестабилност и залитане при ходене.

Какъв антибиотик може да е причинил тези симптоми?

а) ампицилин

б) цефоперазон

в) гентамицин

г) еритромицин

д) линкомицин

010. Посочете антибиотика, който е избраното лекарство

при лечение на инфекции, причинени от стафилокок. ауреус:

а) пеницилин

б) гентамицин

в) азитромицин

г) хлорамфеникол

д) амоксицилин-клавуланат

011. Посочете антибиотика, който е избраното лекарство

при лечение на инфекции,

причинени от метицилин-резистентен стафилокок:

а) линкомицин

б) еритромицин

в) ванкомицин

г) пеницилин

д) оксацилин

012. Посочете антибиотика

а) ампицилин

б) гентамицин

в) цефоперазон

г) метранидазол

д) тетрациклин

013. Активен срещу атипични патогени

(микоплазма, хламидия, легионела):

а) гентамицин

б) еритромицин

в) ампиокс

г) хлорамфеникол

д) клиндамицин

014. Котримоксазол е лекарство на избор при лечение на:

а) Пневмоцистна пневмония при пациенти с имунен дефицит

б) дифтерия

в) холангит

г) пневмококова пневмония

д) амебна дизентерия

015. Кой антибиотик е противопоказан при пациенти

получават мускулни релаксанти или с миастения гравис?

а) ампицилин

б) гентамицин

в) еритромицин

г) линкомицин

д) ципрофлоксацин

016. За болен от бронхиална астма,

постоянно приемане на перорален преднизолон, Teopec, Berotec инхалация,

поради свързана бронхопулмонална инфекция

Предписани са еритромицин и бромхексин.

На третия ден от лечението пациентът разви главоболие, тревожност,

раздразнителност, сърцебиене, усещане за нередности в сърцето,

понижено кръвно налягане, повишена температура, гадене, повръщане.

Тези симптоми са свързани с токсичния ефект на кое лекарство?

а) преднизолон

б) теопек

в) еритромицин

г) Беротек

д) бромхексин

017. Концентрацията на теофилин в кръвта се намалява от всички тези лекарства,

а) фенобарбитал

б) рифампицин

в) карбамазипин

г) нифедипин

д) фенитоин

018. Средната терапевтична концентрация на теофилин в плазмата е:

а) 10-20 µg/ml

б) 25-30 µg/ml

в) 30-35 mcg/ml

г) 35-40 mcg/ml

д) 5-10 µg/ml

019. С комбинираното действие на теофилин и циметидин, действието на аминофилин:

а) засилва се

б) нараства или не се променя

в) намалява

г) намалява или не се променя

г) не се променя

020. Продължителност на действие на ксантините:

а) 1-2 часа

б) 2-3 часа

в) 3-4 часа

г) 6-8 часа

д) 10-12 часа

021. Какъв е механизмът на действие на агонистите на 2-адренергичните рецептори?

а) инхибиране на фосфодиестераза

б) инхибиране на дегранулацията на мастоцитите

в) блокиране на хистаминовите рецептори

г) инхибиране на ефекта на левкотриените върху дихателните пътища

д) активиране на аденилатциклазата, повишено образуване на сАМР

022. Посочете началото, максимума на действие и продължителността на действие

фенотерол (Беротека):

а) незабавно, 10 минути, 6 часа

б) 15 минути, 30 минути, 6 часа

в) 2-3 минути, 20 минути, 2,5 часа

г) 5-10 минути, 30 минути, 6 часа

д) 30-40 секунди, 20 минути, 3-5 часа

023. Посочете най-честите нежелани реакции

инхалаторни форми на глюкокортикоиди:

а) развитие на остеопороза

б) хиперкортицизъм

в) кандидоза на устната кухина и фаринкса

г) артериална хипертония

024. За разлика от беклометазон пропионат, будезонидът има:

а) по-голям афинитет към рецепторите в белите дробове,

претърпява активна биотрансформация в черния дроб

при първото изиграване

б) инхибира в по-голяма степен образуването на хидрокортизон

в) по-често води до развитие на хипергликемия

г) по-често причинява обостряне на бронхопулмонална инфекция

д) няма разлика между лекарствата

025. Посочете дългодействащ 2-агонист:

а) салбутамол

б) тербуталин

в) фенотерол

г) орципреналин сулфат

д) формотерол

026. Посочете лекарството, което има най-силен инхибиращ ефект

върху стомашната секреция:

а) омепразол

б) циметидин

в) фамотидин

г) сукралфат

за лечение на рецидив на язва на дванадесетопръстника:

028. Пациент с ревматоиден полиартрит,

е на НСПВС от дълго време.

Какво лекарство е показано за този пациент?

за предотвратяване на язва?

а) сукралфат

б) гастроцепин

в) ранитидин

г) Маалокс

д) мизопростол

029. При лечението на хеликобактериоза най-ефективната употреба е:

а) ранитидин

б) оксацилин

в) де-нола

г) де-нола + ампицилин (амоксицилин)

д) Маалокс

030. Най-рационалната схема за предписване на антиациди

при пациенти с пептична язва:

а) 20 минути преди хранене

б) веднага след хранене

в) 20 минути след хранене и през нощта

г) един час след хранене и през нощта

д) независимо от храненето 4-5 пъти на ден

031. Какви антихипертензивни лекарства

действат предимно като постганглионарни адренергични блокери?

а) пентамин

б) клонидин

в) гуанетидин сулфат

г) анаприлин

д) хлорталидон

032. Какви антихипертензивни лекарства

действа върху неврохуморалните механизми за регулиране на кръвното налягане?

а) клонидин

б) каптоприл

в) миноксидил

г) гуанетидин

д) хидрохлоротиазид

033. Посочете механизмите на хипотензивното действие на veroshpiron:

а) намаляване на активността на плазмения ренин

б) блокада на адренергичните рецептори

в) намаляване на обема на циркулиращата течност

г) намаляване на общото периферно съпротивление

д) конкурентен алдостеронов антагонист

034. Бионаличността на нифедипин е ниска поради:

а) пресистемно елиминиране в черния дроб

б) ниска абсорбция

в) свързване с плазмените протеини

г) инактивиране в стомашно-чревния тракт

035. Посочете продължителността на хипотензивния ефект на еднократна доза клонидин

при перорален прием:

а) 1-2 часа

б) 6-8 часа

в) 10-12 часа

г) 2-24 часа

г) до 3 дни

036. Облекчаването на неусложнена хипертонична криза трябва да започне:

а) с интрамускулно инжектиране на 1,0 ml 0,01% разтвор на клонидин

б) с 40 mg фуроземид перорално

в) с 10-20 mg нифедипин сублингвално

г) с 40 mg анаприлин перорално

д) с интравенозно 1,0 ml 0,5% разтвор на фентоламин

037. Каптоприл е противопоказан при пациенти:

а) с цироза на черния дроб

б) с хронична бъбречна недостатъчност

в) с cor pulmonale

г) с диабет

г) с пептична язва

038. Посочете механизма на действие на нитроглицерина:

а) блокада на -адренергичните рецептори

б) спазмолитичен, миотропен ефект

върху гладката мускулатура на съдовата стена

в) блокиране на малки калциеви канали в клетъчната мембрана

г) повишена активност на -рецепторите

съдовата стена на коронарните артерии

д) увеличава бавния поток на калций в клетката

039. Какви са противопоказанията за предписване на нитрати?

а) остър миокарден инфаркт

б) артериална хипертония

в) хипотония

г) брадикардия

д) атриовентрикуларен блок

040. На какво се дължи антиангинозният ефект на адренергичните блокери?

а) разширяване на коронарните съдове

б) намаляване на след и преднатоварване на сърцето

в) намаляване на сърдечната функция

г) централен механизъм на действие

д) повишена нужда от кислород на миокарда

041. Посочете селективен адренергичен блокер:

а) обиден

б) тразикор

в) уиски

г) сектален

042. Каква е бионаличността на пропранолол, когато се приема перорално?

043. Каква е бионаличността на верапамил при перорален прием?

044. Посочете противопоказанията за употребата на нифедипин:

а) артериална хипертония

б) сърдечна недостатъчност

в) бронхиална астма

г) артериална хипотония

д) атриовентрикуларен блок 2-ра степен

045. Посочете лекарство, принадлежащо към 1-ва група антиаритмични лекарства

(мембрано стабилизиращ ефект):

а) лидокаин

б) изоптин

в) кордарон

г) хинидин

д) дифинин

046. Посочете лекарство, принадлежащо към 2-ра група антиаритмични лекарства

(местни анестетици):

а) мекситил

б) обиден

в) уиски

г) кордарон

д) новокаинамид

047. Посочете лекарство, принадлежащо към 3-та група антиаритмични лекарства

(-адренергични блокери):

а) лидокаин

б) тразикор

в) кордарон

г) хинидин

д) изоптин

048. Посочете продължителността на действие на лидокаина:

а) 20 минути

б) 60 минути

в) 1,5-2 часа

д) 12 часа

049. Посочете полуживота на кордарон:

а) 4-6 часа

б) 1-2 часа

в) 20-24 часа

050. Посочете времето за достигане на максималната концентрация на изоптин

в кръвната плазма при перорален прием:

а) 10 минути

б) 50 минути

в) 1,5-2 часа

г) 8 часа

д) 10 часа

051. Посочете местната упойка,

с най-изразен антиаритмичен ефект:

а) лидокаин

б) тримекаин

в) ксикаин

г) новокаин

д) мекситил

052. Развитието на толерантност към нитрати зависи главно от:

а) от пътя на въвеждане на нитрати

б) от момента на достигане на максимална концентрация в кръвта

в) върху продължителността на действие

г) от комбинация с други лекарства

д) нито един от горните фактори

не влияе върху развитието на толерантност

053. За разлика от изосорбид динитрата, 5-изосорбид мононитрат:

а) не претърпява първично пресистемно елиминиране

докато преминава през черния дроб

б) не предизвиква развитие на толерантност

в) не предизвиква метхемоглобиния

г) не предизвиква главоболие

д) няма разлики между лекарствата

054. Посочете дългодействащо противовъзпалително средство:

а) аспирин

б) аналгин

в) пироксикам

г) индометацин

д) ортофен

055. При приемане на салицилати се наблюдава най-ниската им концентрация:

а) в бъбреците

б) в черния дроб

в) в миокарда

г) в белите дробове

г) в мозъка

056. Посочете глюкокортикоидните лекарства с удължено действие:

а) преднизолон

б) полкортолон

в) дексаметазон

г) кеналог

д) метилпреднизолон

057. Посочете страничния ефект на никотиновата киселина:

а) липодистрофия

б) хиперурикемия

в) рабдомиоза

г) зрително увреждане

д) бронхоспазъм

058. Посочете страничните ефекти на секвестрантите на жлъчните киселини:

а) сърбеж по кожата

б) диария

в) запек

г) зрително увреждане

д) депресия

059. Кой антихистамин е противопоказан

а) супрастин

б) пиполфен

в) дифенхидрамин

г) тавегил

д) фенкарол

060. Кой антихистамин е противопоказан

при лечение на алергични реакции към прилагане на лекарства,

а) пиполфен

б) супрастин

в) дифенхидрамин

г) тавегил

д) фенкарол

061. Полуживотът на лекарствата е:

а) време за достигане на максимална концентрация на лекарството в плазмата

б) времето, през което лекарството достига системното кръвообращение

в) времето, през което лекарството се разпределя в организма

г) време, през което концентрацията на лекарството в плазмата намалява с 50%

д) време, през което половината от приложената доза достига до прицелния орган

062. Терапевтичният индекс е:

а) терапевтична доза на лекарството

б) съотношението на концентрацията на лекарството в орган или тъкан

на концентрацията му в кръвната плазма

в) диапазонът между минимум и максимум

г) процент на лекарството, несвързано с протеина

д) диапазон между минимум и максимум

терапевтични концентрации на лекарства

063. Рецепторните агенти на конкурентното действие включват:

а) нестероидни противовъзпалителни средства

б) - адренергични блокери

в) бримкови диуретици

г) нитрати

д) флуорохинолони

064. При предписване на следните лекарства

Трябва да се вземе предвид функцията както на черния дроб, така и на бъбреците:

а) липофилни, образуващи неактивни метаболити

б) липофилни, образуващи активни метаболити

в) хидрофилен

г) хепатотоксичен

д) нефротоксичен

065. Избирателността на действието на лекарственото вещество зависи от:

а) от полуживота

б) относно начина на приложение

в) от връзката с протеина

г) върху обема на разпространение

д) в зависимост от дозата

066. Следните групи нежелани реакции са строго дозозависими:

а) фармацевтични

б) токсичен

в) алергични

г) мутагенен

д) синдром на отнемане

067. Избройте групи лекарства с тесен терапевтичен индекс:

а) - блокери

б) пеницилини

в) сърдечни гликозиди

г) метилксантини

д) мощни диуретици

068. Лекарство на избор при наличие на атипични патогени

(микоплазма, хламидия) е:

а) еритромицин

б) метронидазол

в) гентамицин

г) карбеницилин

д) цефуроксим

069. Лекарства на избор при наличие на атипични патогени

(микоплазма, хламидия) са:

а) макролиди

б) пеницилини

в) аминогликозиди

г) цефалоспорини

д) сулфонамиди

070. Посочете антибактериалното лекарство,

с най-голяма антианаеробна активност:

а) еритромицин

б) ампицилин

в) тетрациклин

г) гентамицин

д) цефотетан

071. Чревна дисбиоза

причиняват всички от следните антибактериални лекарства, с изключение на:

а) полусинтетични лекарства

б) тетрациклини

в) флуорохинолони

г) перорални цефалоспорини

д) макролиди

072. Следните антибактериални лекарства са нефротоксични:

а) гентамицин

б) карбеницилин

в) еритромицин

г) цефазолин

д) ванкомицин

073. Посочете антибактериално лекарство,

не е активен срещу пневмококи:

а) азитромицин

б) пеницилин

в) цефтриаксон

г) ципрофлоксацин

д) хлорамфеникол

074. Изберете комбинация от антибактериални лекарства,

със синергично действие и безопасност:

а) пеницилини + тетрациклини

б) пеницилини + цефалоспорини

в) пеницилини + макролиди

г) пеницилини + аминогликозиди

д) пеницилини + сулфонамиди

075. Прониква добре през кръвно-мозъчната бариера

следните антибактериални лекарства:

а) пеницилини

б) макролиди

в) тетрациклини

г) аминогликозиди

д) цефалоспорини

076. Лекарството по избор при лобарна пневмония е:

а) цефаклор

б) доксициклин

в) метицилин

г) цефотаксим

д) пеницилин

077. Лекарството на избор при фарингит е:

а) цефаклор

б) тетрациклин

в) цефтазидим

г) офлоксацин

д) пеницилин

078. Ново поколение макролидни антибиотици

има следните предимства:

а) висока бионаличност

б) широк спектър на антибактериално действие

в) бактерициден ефект

г) дълъг полуживот

д) път на екскреция през бъбреците

079. Флуорохинолоните се различават от хинолоните по следните свойства:

а) широк антибактериален спектър на действие

б) бактериостатичен ефект

в) високо проникване в тъканите

г) пост-антибактериален ефект

г) перорално приложение

080. Изберете лекарство,

максимално потискане на секрецията на солна киселина:

а) пирензепин

б) циметидин

в) карбеноксолон

г) антиациди

д) омепразол

081. Максималният брой странични ефекти сред Н2 блокерите

обаждания:

а) циметидин

б) роксатидин

в) низатидин

г) ранитидин

д) фамотидин

082. Синтетични аналози на простагландини (енпростил, мизопростол)

причиняват следните ефекти:

а) антисекреторен ефект

б) секреция на барбитурати

в) образуване на слуз

г) репаративно действие

083. Инхибира метаболизма на други лекарства:

а) омепразол

б) карбеноксолон

в) циметидин

г) фамотидин

д) гастроцепин

084. Продължителността на антисекреторното действие на омепразол е:

а) 2-4 часа

б) 8-10 часа

в) 16-20 часа

д) 3 дни

085. Посочете лекарството,

който е мощен стимулатор на образуването на слуз в стомаха:

а) карбеноксолон

б) платифилин

г) омепразол

д) метоклопрамид

086. При наличие на бъбречна недостатъчност

Необходима корекция на дозата на лекарството:

а) аналози на простагландин

б) омепразол

в) H2 блокери

г) сукралфат

д) антихолинергици

087. Посочете антисекреторно лекарство, което блокира "протонната помпа"

а) метоклопрамид

б) карбеноксолон

в) пирензепин

г) сукралфат

д) омепразол

088. При бъбречна патология

Настъпват следните промени във фармакогенетиката на лекарствата:

а) нарушена бъбречна екскреция

б) повишаване на концентрацията на лекарства в кръвната плазма

в) намалено свързване с плазмените протеини

г) увеличаване на полуживота

д) намалена бионаличност

089. Чернодробната цироза причинява следните промени във фармакокинетиката на лекарствата:

а) намаляване на метаболизма при първо преминаване

б) намалено свързване с плазмените протеини

в) увеличаване на полуживота

г) повишена бионаличност

д) намаляване на обема на разпределение

090. При сърдечна недостатъчност

Наблюдават се следните промени във фармакокинетиката на дигоксин:

а) намаляване на абсорбцията в стомашно-чревния тракт с 30%

б) намалено свързване с плазмения протеин

в) повишен метаболизъм в черния дроб

г) намалена бъбречна екскреция

д) увеличаване на полуживота

091. Алкохолът води до:

а) за увеличаване на абсорбцията на лекарството

в) за забавяне на метаболизма в черния дроб

г) до намаляване на бъбречната екскреция

д) за увеличаване на полуживота

092. Никотинът води до:

а) за намаляване на абсорбцията на лекарството

б) да се увеличи обемът на разпространение на лекарствата

в) за увеличаване на връзката с плазмените протеини

г) за ускоряване на метаболизма в черния дроб

д) за увеличаване на бъбречната екскреция на лекарства

093. При стенокардия при усилие е показано:

а) нифедипин

б) пропранолол

в) качулка

г) еналаприл

д) клонидин

094. За ангина на Prinzmetal (вазоспастична) е показано следното:

а) нифедипин

б) обиден

в) дипиридамол

г) допегит

д) каптоприл

095. Критерият за ефективност на антиангинозно лекарство е:

а) увеличаване на времето за зареждане на VEM >1 минута

б) увеличаване на количеството консумиран NTG

в) увеличаване на HEM времето - проби >2 минути

г) намаляване на времето за зареждане

д) преход на пациента от 2-ри към 3-ти функционален клас на ангина пекторис

096. Антиангиналните лекарства включват:

а) камбанен звън

б) качулка

в) аспирин

г) верапамил

097. За облекчаване на пристъп на ангина използвайте:

а) сутак

б) нитронг

в) нитроглицерин

г) верапамил

д) дилтиазем

098. От антиангинални лекарства

с комбинация от коронарна артериална болест и артериална хипертония е показано следното:

а) сутак

б) верапамил

в) каптоприлил

г) звънец

д) еналаприл

099. Толерантността е най-вероятно да се развие, когато се използва:

а) тринитролонг

б) сустака

в) сублингвален нитроглицерин

г) изосорбитол-5-мононитрат

д) нитронга

100. Методът за проследяване на ефективността на антиангинозната терапия е:

а) Холтер ЕКГ мониторинг

б) контрол на нивата на кръвните липиди

в) 24-часово мониториране на артериалното налягане

г) измерване на дихателната функция (функция на външното дишане)

д) измерване на артериалното налягане при орто- и клиностаза

101. Лекарството на избор при ангина пекторис при пациент с брадикардия е:

а) пиндолол

б) пропранолол

в) верапамил

г) дилтиазем

д) метопролол

102. Лекарство на избор при ангина пекторис

при пациент със сърдечна недостатъчност е:

а) верапамил

б) Коринфар

в) дилтиазем

г) ацебуталол

д) нитросорбитол

103. Инхалаторните кортикостероиди включват:

а) хидрокортизон

б) беклометазон

в) преднизолон

г) полкортолон

д) дексаметазон

104. Селективните дългодействащи 2-агонисти включват:

а) флутиказон

б) салметерол

в) салбутамол

г) фенатерол

д) тербуталин

105. За облекчаване на пристъп на бронхиална астма се използва:

а) ипратропиев бромид

б) теопек

г) салбутамол

д) задитен

106. Мембранен стабилизатор за орално приложение е:

а) кетотифен

б) недокрамил натрий

в) натриев хромогликат

г) супрастин

д) ипратропиев бромид

107. За синдрома на "заключване" се използва следното:

а) салбутамол

б) фенотерол

в) теопек

г) аминофилин

г) адреналин

108. Муколитичните средства включват:

а) кодеин

б) натриев хромогликат

в) ацетилцистеин

г) салметерол

д) теофилин

109. Когато се използват едновременно

повишава концентрацията на теофилин в кръвта:

а) офлоксацин

б) пеницилини

в) цефтриаксон

г) гентамицин

д) бисептол

110. Когато се използват едновременно

намалява концентрацията на теофилин в кръвта:

а) пефлоксацин

б) циметидин

в) рифампицин

г) еритромицин

д) ампиокс

111. При повишено налягане в белодробната артерия

при пациент с бронхиална астма е показано следното:

а) верапамил

б) нифедипин

в) дигоксин

д) беклометазон

112. При бронхиална астма на фона на хроничен бронхит е показано:

а) ипратропиев бромид

б) адреналин

в) ефедрин

г) кетотифен

г) супрастин

113. Посочете предпочитания начин на приложение на лекарството

за застойна сърдечна недостатъчност:

а) ректално

б) сублингвални

в) вътре

г) венозно

г) кожна

114. Избройте лекарствата,

имащи директен положителен инотропен ефект:

а) дигоксин

б) допамин

в) норепинефрин

г) аминофилин

д) хидралазин

115. Посочете държавите,

повишена чувствителност към сърдечни гликозиди:

а) старост

б) тиреотоксикоза

в) белодробно сърце

г) хипокалиемия

д) застойна сърдечна недостатъчност

116. Избройте лекарствата, които взаимодействат с

Концентрацията на дигоксин в кръвта може да се увеличи:

а) фосфулагел

б) хинидин

в) верапамил

г) амиодарон

117. Избройте факторите, които забавят абсорбцията на сърдечните гликозиди

от стомашно-чревния тракт:

а) хронична бъбречна недостатъчност

б) застойна сърдечна недостатъчност

в) стомашна язва

г) комбинирана употреба с антиациди

118. Избройте факторите

осигуряване на най-голяма безопасност и ефективност на диуретиците

за продължително лечение на сърдечна недостатъчност:

а) максимални дози

б) средни дози

в) минимални дози

г) дневен прием

г) периодична употреба

119. Посочете най-ефективния диуретик

за лечение на застойна сърдечна недостатъчност

с развитието на вторичен хипералдостеронизъм:

а) етакринова киселина (урегид)

б) хлорталидон (хигротон)

в) ацетазоламид (диакарб)

г) спиронолактон (верошпирон)

г) триампур

120. Посочете основния терапевтичен ефект на нитросорбида

при пациенти със сърдечна недостатъчност:

а) разширяване главно на артериолите и намаляване на следнатоварването

б) разширяване предимно на венули и намаляване на предварителното натоварване

в) директен положителен инотропен ефект

г) повишена диуреза и намалено преднатоварване

121. Избройте лекарства, които имат антихолинергични странични ефекти:

а) лидокаин

б) хинидин

в) амиодарон (кордарон)

г) верапамил

д) прокаинамид (новокаинамид)

122. Избройте групите антиаритмични лекарства,

имащи антифибрилиращ ефект:

а) сърдечни гликозиди

б) калциеви антагонисти (4-та група)

в) -блокери (2-ра група)

г) амиодарон, бретилиев тозилат (група 3)

д) хинидин, прокаинамид и други лекарства от група 1а

123. Избройте лекарствата,

което може да провокира пристъп на предсърдна тахиаритмия

за синдром на Волф-Паркинсон-Уайт:

а) дигоксин

б) амиодарон

в) верапамил

г) пропранолол

д) етмозин

124. Посочете лекарства, които имат аритмогенен ефект:

а) аймалин

б) мексилетин

в) пропафенон

г) амиодарон

125. Избройте показанията за лечение с антиаритмични лекарства:

а) аритмии, причиняващи хемодинамични нарушения

б) субективна непоносимост към аритмия

в) нарушение на ритъма на високи градации

г) чести ритъмни нарушения

126. Избройте лекарствата,

основният антиаритмичен ефект на който

свързани с удължаване на атриовентрикуларната проводимост:

а) пропранолол

б) лидокаин

в) верапамил

г) дигоксин

д) прокаинамид

127. Посочете ефектите от взаимодействието между амиодарон и дизопирамиди

(ритмичен, норвеж):

а) забавяне на метаболизма на дизопирамид

б) забавяне на метаболизма на амиодарон

в) повишен риск от странични ефекти на дизопирамид

г) повишен риск от странични ефекти на амиодарон

128. Посочете страничния ефект на нифидепин:

а) брадикардия

б) бронхоспазъм

в) подуване на краката и стъпалата

г) развитие на F-V блокада

д) улцерогенност

129. Посочете лекарството,

механизъм на хипотензивно действие на който

е блокада на -рецептори:

а) клонидин

б) празозин

в) пропранолол

г) качулка

д) верапамил

130. Лекарство по избор

при пациент с хипертония и застойна циркулаторна недостатъчност

е:

а) еналаприл

б) нитросорбид

в) клонидин

г) аделфан

д) пентамин

131. Оптималното лекарство за дългосрочна антихипертензивна терапия трябва да:

а) влияят на метаболизма

б) имат реакции на отскок

в) имате симптоми на отнемане

г) имат стабилна концентрация в кръвта

д) предизвикват ортостатични реакции

132. Избройте групите антихипертензивни лекарства,

намаляване на активността на системата ренин-ангиотензин-алдостерон:

а) АСЕ инхибитори

б) -блокери

в) централни агонисти

г) тиазидни диуретици

д) калциеви антагонисти

133. Посочете антихипертензивни лекарства,

които трябва да се използват с повишено внимание

с комбинация от захарен диабет и хипертония:

а) верапамил

б) пропранолол

в) дилтиазем

г) хипотиазид

д) еналаприл

134. Провеждане на лекарствен мониторинг

се изисква при лечение на следните групи лекарства:

а) антиконвулсанти

б) 2-симпатикомиметици

в) метилксантини

г) глюкокортикоиди

д) М-холиномиметици

135. Развитието на асистолия е възможно при комбиниране на пропранолол:

а) с фенобарбитал

б) с фуроземид

в) с верапамил

г) с фенитеин

д) с циметидин

136. Рискът от токсични ефекти се увеличава при комбинацията на гентамицин:

а) с фуроземид

б) с пеницилин

в) с метилксантини

г) с макролиди

д) с глюкокортикоиди

КЛИНИЧНА ФАРМАКОЛОГИЯ

114 - a, b, c, d, d

115 - a,b,c,d,e

088 - а,б,в,г

089 - а,б,в,г

090 - а, б, г, г

124 - a,b,c,d,e

078 - а,б,в,г

079 - а,в,г,г

083 - а,б,в,г

Изберете правилното твърдение: а) бионаличността е количеството лекарство, навлизащо в системното кръвообращение, изразено като процент от приложената доза, б) бионаличността се определя от степента на абсорбция на лекарството в стомашно-чревния тракт и тежестта на ефекта на първото преминаване през черния дроб в) бионаличността се определя по формулата: F = AUC (им. или перорално)/AUC (i.v.) г) бионаличността на лекарството при интрамускулно приложение се определя от степента на неговата абсорбция и биотрансформация в организма .
Отговор: a B C

2.
Отговор: Атровент

3.

Отговор: а, г

4.

Отговор:

5.
Отговор:

6.

д) Ксилитол
Отговор: a,c

7.

Отговор: а,б,г

8.
Отговор:

9. е приет с остър миокарден инфаркт, настъпил преди 5 часа Предписания: анаприлин 20 mg 4 пъти на ден перорално, хепарин интравенозно 10 000 единици на всеки 4 часа В същото време е възможно да се увеличи времето за съсирване на кръвта до 18-23 минути . На следващия ден той беше диагностициран с дясностранна пневмония на долния лоб.Бензилпеницилин натриева сол (1 000 000 единици на всеки 4 часа) беше предписан интравенозно.След 4 часа времето за съсирване на кръвта беше 8 минути. Каква е вашата тактика?
Отговор:

10.

11.
Отговор: Вит.В12 в доза 500 mcg/ден през ден, фолиева киселина в доза 1,5 mg/ден, железен сулфат (80 mg Fe2+) веднъж дневно

12.

Отговор: Вит.С

13.

Отговор: Церебролизин

14.
алергии (към бутадион, хепарин, метиндол, пеницилин, теофилин).В болницата са предписани реопирин 5 ml IM веднъж дневно, хидрокортизон хемисукцинат 100 mg в кухината на коленните стави, tavegil 0,001 g 2 пъти на ден. След 3 дни b-no разви сърбящи еритематозни обриви по кожата на торса Каква е най-вероятната причина
влошено състояние?
Отговор:

15.



Отговор: a, b, d, f, h, i

16.
Отговор: След няколко месеца

17.


Отговор: a, b, c, d, f

18.
Отговор: a, b, c, d, e, g, h

19.
Отговор:

20.
Отговор: Ципрофлоксацин

21. Феноменът на първото преминаване на лекарството през черния дроб зависи от: а) кръвоснабдяването на черния дроб, б) свързването на лекарството с протеина, в) активността на хепатоцитните ензими, г) нивото на екскреция на лекарството, д) скоростта на абсорбция
Отговор: а, в

22. Лекарства, повлияващи микрозомалните чернодробни ензими: индуктори на микрозомалните чернодробни ензими: а) пеницилин, б) нитроглицерин, в) фенобарбитал, г) фуроземид, д) бутадион, е) кортизол, ж) пропранолол, з) циметидин, i) хлорамфеникол, k ) дифенин
Отговор: c,d

23. е приета в отделението с болки в дясната млечна жлеза, Т. е повишена до 39,5 С. ​​Заболяла е преди 3 дни, на 10-тия ден след раждането. При постъпване в отделението се установява хиперемия на кожата и масивен инфилтрат с флуктуация в центъра в горно-външен квадрант на дясна млечна жлеза.Диагноза: остър десностранен мастит.Б. Взета е култура на секреция от рана Определете антибиотика на първи избор.
Отговор: Цефазолин

24.

Отговор: Анафилактична реакция

25.

Отговор: Левомицетин

26.
Отговор:

27.
Отговор: Бигуаниди

28.

Отговор: Хипотония, световъртеж.

29.

Оценете действията на лекаря.

30. Пациент Д., 53 г., с диагноза исхемична болест на сърцето, стабилна стенокардия??? FC, постинфарктна кардиосклероза, предсърдно мъждене, CNC??B st. Приема строфантин, дигоксин, фуроземид и панангин в средни терапевтични дози. Неочаквано температурата на пациента се повиши до 38,4 ° C, появиха се кашлица, задух и крепитус в белите дробове вдясно. Рентгенова снимка на белите дробове вдясно в долния лоб разкрива зона на инфилтрация. Към лечението са добавени гентамицин, сулфокамфокаин и супрастин.

Отговор:

31.

Отговор: Фентоламин.

32.

Отговор: d,d

33.

Отговор:

34.
Отговор:

35.
Отговор: Еналаприл.

36.

Отговор: а,б,г

37.

Отговор:

38.
Отговор:

39.
Отговор:

40.

Отговор: Всички изброени

41. Лекарства, повлияващи чернодробните микрозомални ензими: инхибитори на чернодробните микрозомални ензими:
а) пеницилин, б) нитроглицерин, в) фенобарбитал,
г) фуроземид, д) бутадион, е) кортизол, ж) пропранолол,
h) циметидин, i) хлорамфеникол, j) дифенин
Отговор: h,i

42.
Отговор: След 7-14 дни

43. Посочете комбинация от лекарства, която води до конкуренция за свързване с протеини, което може да доведе до опасно повишаване на съдържанието на свободната фракция на едно от лекарствата в кръвта и появата на симптоми на предозиране:
Отговор: неодикумарин и бутадион

44. Изберете лекарство с тесен терапевтичен диапазон:
а) пеницилини, б) антиконвулсанти,
в) антиаритмични лекарства, г) дигоксин, д) метотрексат, е) теофилин, ж) циклоспорин, з) макролиди
Отговор: b,c,d,e,f,g

45. Посочете комбинации от лекарства, при които поради конкуренция за свързване с протеина се наблюдава повишаване на концентрацията в кръвната плазма на свободната фракция на едно от тях: а.строфантин и мисклерон, б.дигитоксин и мисклерон, в.неодикумарин. и бутадион, г. нифедипин и хидрохлоротиазид
Отговор: b,c

46. Установени са чести камерни екстрасистоли и пароксизми на предсърдно мъждене.Сърдечна честота 74 в минута, кръвно налягане 140/80 mmHg.През последните 3 години ни тормозят пристъпи на ангина пекторис на усилие и покой.Лечението е проведено с кордарон. Като се вземат предвид страничните ефекти на предписаното лекарство, изберете лекарство за по-нататъшно лечение b -nogo: а) хинидин,
б) Бонекор, в) Етацизин,

Отговор: а, б

47. Известно е, че при комбинирането на хинидин и дигоксин често се наблюдава гликозидна интоксикация, с какво е свързана? Фармакодинамично взаимодействие:
Отговор: синергия

48. Известно е, че при комбиниране на хинидин и дигоксин се наблюдава гликозидна интоксикация, с какво е свързана? Фармакокинетично взаимодействие, ефект на хинидин върху:
Отговор: свързване с протеини

49. Критични периоди на вътрематочно развитие:
А. период на предимплантационно развитие (1 седмица)
b. Етапът на ембриогенеза завършва до 8 седмици.
V. Етапът на ембриогенеза завършва до 8 месеца.
г. периода непосредствено преди раждането
Отговор: а,б,г

50. Изберете от следните лекарства, които имат изброените свойства: Антимикробни лекарства, чиято употреба е практически безопасна по време на бременност: а) сулфонамиди, включително бисептол,
б) аминогликозиди, тетрациклини, рифампицини, метронидазол (през 1 триместър на бременността), в. пеницилини, цефалоспорини, еритромицин, линкомицин, фузидин, г. антимикотични средства, противотуморни
антибиотици.
Отговор: V

51. Метронидазол се предписва на кърмеща майка, посочете страничните ефекти:
а) повишена възбудимост, тахикардия, б) потискане на апетита, повръщане, в) потискане на централната нервна система, дишане, загуба на телесно тегло, г) повишена секреция на пролактин, набъбване на млечните жлези, е. надбъбречна хипоплазия, метаболитни нарушения, повишен риск от развитие на билирубинова енцефалопатия, например кръвоизливи, дихателна недостатъчност, ацидоза, потискане на хематопоезата, анемия, недохранване, дисбактериоза.
Отговор: b

52. Антимикробни лекарства на първи избор при новородени: а. бензилпеницилин, оксацилин, карбеницилин, гентамицин, амикацин, б. бензилпеницилин, оксацилин, бицилини, цефазолин, цефотаксим, еритромицин, линкомицин, нистатин, в. карбеницилин, гентамицин, сизомицин, амикацин, тобрамицин , зепорин (ако цефалоспорините от първо поколение са неефективни), еритромицин, линкомицин, нистатин, леворин, карбеницилин,
гентамицин, сизомицин
Отговор: b

53.
Отговор:

54. Основни характеристики на фармакокинетиката на лекарството при хора в напреднала възраст:
а.намаляване на скоростта на абсорбция, б.ускоряване на абсорбцията, в.намаляване на скоростта на разпределение, г.ускоряване на разпределението, д.намаляване на свързването на лекарствата с плазмените протеини, е.увеличаване на свързването на лекарствата към плазмените протеини, g.забавяне на метаболизма, h.ускоряване на метаболизма,
и. забавяне на елиминирането на лекарствата, к. ускоряване на елиминирането на лекарствата.
Отговор: a,c,d,g,i

55.
Отговор: b,c,d

56. Посочете страничните ефекти на бета-блокерите: а) брадикардия, б) артериална хипотония, в) бронхоспазъм, г) тахикардия, д) дисфункция на щитовидната жлеза,
е) интермитентно накуцване, ж) AV блок
Отговор: a, b, c, f, g

57.
състояния: а) Естествен ход на заболяването, б) Развитие на толерантност към нитрати, в) Синдром на междукоронарна кражба, г) Поява на синдром на отскачане д) феномени на идиосинкразия
Отговор: а, б

58. Посочете страничните ефекти на амиодарон: а) брадикардия, б) артериална хипотония, в) бронхоспазъм, г) тахикардия, д) дисфункция на щитовидната жлеза, е) интермитентно накуцване, ж) AV блок
Отговор: a,c,d,g

59. Как ще се промени вашата антиангинозна терапия, ако пациентът развие мозъчен инсулт, докато е на нитратна терапия?
Отговор: отмяна на нитрати и предписване на антиангинозно лекарство от друга група

60. Кои антихипертензивни лекарства се считат за най-безопасни за пациенти в напреднала възраст: а) бета-блокери, б) ганглийни блокери, в) симпатиколитици, г) бавни блокери на калциевите канали, д) тиазидни
диуретици, д) АСЕ инхибитори.
Отговор: d,d

61. Режим на лечение с кордарон:
Отговор: по схема, включваща постепенно намаляване на дозата от 600 mg до 200 mg на ден

62. Как МАО инхибиторите (антидепресантите) влияят върху пресорния ефект на директните и индиректните адренергични стимуланти?
Отговор: засилване на ефекта

63. Небензодиазепинов бензодиазепинов рецепторен агонист:
Отговор: Золпидем

64. Хипнотичен - съединение от алифатната серия:
Отговор: Хлорал хидрат

65.

Отговор: А (б)

66. Протамин сулфат се предписва при предозиране:
Отговор: Хепарин

67. Кой метод за детоксикация е най-ефективен при отравяне с вещества, които се свързват с протеини и кръвни липиди?
Отговор: Хемосорбция

68. Принципът на действие на налоксон при остро отравяне с морфин:
Отговор: Пречи на ефекта на морфина върху опиоидните рецептори

69. Посочете лекарства, които имат антиоксидантни свойства: а) верапамил б) вит.А, в) вит.К, г) вит.С, д) вит.Е, е) селен, ж) карнозин, з) доксициклин
Отговор: b, d, e, f, g

70. Какви ефекти са характерни за антипсихотиците?
а) антипсихотик, б) седатив, в) антиеметик
Отговор: a B C

71. B-oh, на 64 години, разви остър пристъп на закритоъгълна глаукома със силна болка в дясното око, излъчваща се към главата. Появиха се гадене и повръщане, задух, признаци на хипертонична криза тип 2 със сърдечна честота 62.
при мин. АН 200/140 mmHg Б страда от много години хипертония Има голям брой влажни фини хрипове в белите дробове Какви диуретици са показани на пациента? А. Клопамид, б.Верошпирон, в. Хипотиазид, г. Фуроземид IV, г. Диакарб:
Отговор: d,d

72. страда от захарен диабет в продължение на 15 години, за което получава инсулин от 70 единици / ден, което поддържа гликемичното ниво в рамките на 7,5-8,6 mmol / l. Напоследък кръвното налягане започва да се повишава до
170/90-180/100 mmHg, поради което лекуващият лекар предписа обзидан в дневна доза от 120 mg Какви странични ефекти трябва да се очакват при тази комбинация от лекарства? а. Хипергликемия до кома, б. Сърдечна недостатъчност, в. Хипогликемия до кома, г. Ортостатична хипотония, д. Хипертония
Отговор: b,c

73. страдащи от хормонално-зависима бронхиална астма, е предписан преднизолон (5 mg дневно), салбутамол (вдишване на 2 дози аерозол 4 пъти на ден).Поради прояви на конвулсивен синдром (анамнеза за травматично мозъчно увреждане) е предписан фенобарбитал. седмица по-късно се разви екзацербация на бронхиална астма, с какво е свързано това?
А. Фенобарбитал ускори биотрансформацията на: а. салбутамол, б. преднизолон, Б. Фенобарбитал ускори екскрецията на: а. салбутамол, б. преднизолон, В. Фенобарбитал забави екскрецията на: а. салбутамол, б. преднизолон, D. Фенобарбитал забавя биотрансформацията на: а.салбутамол б.преднизолон
Отговор: А (б)

74. страда от коронарна артериална болест, ангина пекторис FC III Сърдечна честота 90 в минута, кръвно налягане 150/80 mm Hg Анамнеза за хроничен бронхит с бронхоспастичен синдром в ремисия Мастна чернодробна дегенерация Посочете групите лекарства (втори етап на избор лекарствена терапия), оптимална за антиангинална терапия. а.Нитрати и верапамил, б. Нитрати и атенолол
в) Нитрати и анаприлин, г) Нитрати и нифедипин,
д) Нифедипин и амиодарон
Отговор: А

75. за ангина пекторис, приема нитросорбид 10 mg 4 пъти на ден, сърдечна честота 80 в mi.BP 140/80 mm Hg. 1 месец след началото на терапията, пристъпите на стенокардия отново зачестиха. Какви са възможните причини за влошаване
състояния: а) Естествен ход на заболяването, б) Развитие на толерантност към нитрати, в) Синдром на междукоронарна кражба, г) Поява на синдром на отскок, д) феномени на идиосинкразия
Отговор: а, б

76. пристъпи на ангина пекторис се отбелязват при умерено физическо натоварване Има анамнеза за колаптоидно състояние след еднократна доза сублингвален нитроглицерин (оттогава не е приемал нитроглицерин). Съпътстващи заболявания - хипертония (работно кръвно налягане 160/100 mm Hg.
Чл., хипофункция на щитовидната жлеза.Кръвното налягане към момента на изследването е 190/100 mm Hg, сърдечна честота 72 в минута.Пациентът е противопоказан:
Отговор: Амиодарон

77. за артериална хипертония в стадий 2 получава 0,000075 g клонидин 4 пъти на ден.Поради развитието на сенилна депресия е предписан мелипрамин.3 дни след предписването на мелипрамин пациентът развива хипертонична криза.Ден преди това пациентът не е приемал клонидин Какви са възможните причини за влошените условия: а) Последица от естествения ход на заболяването, б) Последица от хипертензивния ефект на мелипрамин, в) Последица от нежелани лекарствени взаимодействия, г) Последица от евентуално спиране на прием на лекарства и развитие на синдром на отнемане.
Отговор: b,c,d

78. при хипертонична криза се прилага интравенозно натриев нитропрусид в големи дози (със скорост 8 mcg/min). Появиха се недостиг на въздух, акроцианоза, натискаща болка зад гръдната кост, мускулни потрепвания Каква е причината за влошаване на състоянието на пациента?
Отговор: Токсични ефекти на цианида

79. Установени са чести камерни екстрасистоли и пароксизми на предсърдно мъждене.Сърдечна честота 74 в минута, кръвно налягане 140/80 mmHg.През последните 3 години ни тормозят пристъпи на ангина пекторис в покой и усилие.Лечението е проведено с кордарон. Като имате предвид страничните ефекти на предписаното лекарство, изберете лекарство
за по-нататъшно лечение на b-nogo: а) хинидин, б) бонекор, в) етацизин,
г) мекситил, д) верапамил, е) пропранолол
Отговор: а, б

80. пароксизмална суправентрикуларна тахикардия на фона на WPW синдром Ajmaline е избран за спиране на атаката Определете оптималния режим на лечение с избраното лекарство: а) 1 mg / kg IV за повече от 10 минути, повторете след 30 минути, ако е необходимо, б) 50 mg IV поток-
3-5 минути в 10 ml 5% разтвор на глюкоза или изотоничен разтвор на NaCl или мускулно, в) 0,5-1 g интравенозно на всеки 2 минути, 0,1-0,2 g или се прилага интрамускулно
г) след парентерално приложение предписвайте 100 mg перорално 4-5 пъти на ден, поддържаща доза 50 mg 3-4 пъти на ден
Отговор: а, г

81. 28-годишен пациент с диагноза СЛЕ поради хронична бъбречна недостатъчност развива отоци на краката и увеличен черен дроб.При ехокардиографско изследване се установява намаление на сърдечния дебит. Сърдечна честота 95/мин, артериално налягане 170/100 mmHg Кои сърдечни гликозиди са показани на пациента?
Отговор: Дигитоксин

82. Пациент на 28 години с диагноза СЛЕ поради хронична бъбречна недостатъчност развива отоци на краката и увеличен черен дроб.При ехокардиографско изследване се установява намаление на сърдечния дебит.Сърдечна честота 95/мин., АН 170/100 mmHg. Пациентът приема дигитоксин.Поради появата на конвулсивен синдром е предписан допълнително фенобарбитал (0,3 g/ден).Кога ще настъпят промени в състоянието на пациента, ако има влияние?
Отговор: След 7-14 дни

83. 57-годишен мъж с постинфарктна атерокардиосклероза и застойна сърдечна недостатъчност степен 2 получава 40 mg фуроземид IV и 300 mg
veroshpiron перорално Каква диуретична терапия ще предпише на пациента в случай на рефрактерност?
Отговор: Фуроземид 80 mg IV и спиронолактон 300 mg перорално

84. страда от неатопична бронхиална астма, придружена от профузна бронхорея Пулс 62 в минута Кръвно налягане 140/80 mmHg Кои лекарства са за предпочитане?
Отговор: Атровент

85. постоянно рецидивиращ синдром на бронхиална обструкция с намалена чувствителност към холин и адренотропни лекарства Страда от бронхиална астма повече от 10 години Какво може да се предпише за намаляване на честотата и тежестта на пристъпите на бронхиална астма: а) Бета инхалация
2-адренергични стимуланти повече от 6 пъти дневно, б) Инхалация на m-антихолинергичен блокер, в) Подкожно приложение на адреналин в по-висока от обичайната доза за облекчаване на бронхоспазма, г) Еуфилин IV, д) Инхалаторни глюкокортикоиди.
Отговор: d,d

86. беше приет с киселини, болки в епигастралната област на гладно, облекчени от приема на натриев бикарбонат FEGDS разкри язва (0,5 см в диаметър) в ампула от 12 бр рН-метрия на стомашен сок: образуване на киселина
заядлива функция със средна интензивност с ниски алкални резерви, холинергичен тип рецепция. Диагноза: пептична язва 12 p.k в остър стадий. Изберете най-ефективното и безопасно лекарство и определете неговия режим на дозиране:
Отговор: Пирензепин преди хранене, 0,05 g 3 пъти на ден в продължение на 2 дни, след това 0,05 g 2 пъти на ден

87. Установена е дискинезия на жлъчния мехур от хипертоничен тип. Изберете най-добрия вариант за лечение.
Отговор: Но-шпа по 1-2 таблетки 3 пъти на ден, отвара от безсмъртниче по 1/2 чаша 30 минути преди хранене.

88. страда от хроничен холецистопанкреатит в продължение на 5 години.През последната седмица след прекъсване на диетата той отбеляза повишена болка в десния квадрант, гадене, горчивина в устата.Изберете най-ефективните холеретични средства, които едновременно имат антимикробна активност:
а) Алохол, б) Холензим, в) Никодин, г) Отвара от вратига, д) Ксилитол
Отговор: a,c

89. Взех 20 таблетки феназепам за суицидни цели.2 часа след приема на лекарството ме закараха в болницата. Б е в съзнание, но рязко потиснат Извършена е стомашна промивка Изберете най-оптималните лаксативи: а) глауберова сол, б) магнезиев сулфат, в) екстракт от кора на зърнастец, г) бисакодил,
д) рициново масло, е) морски водорасли, ж) вазелиново масло
Отговор: а,б,г

90. Мъж на 46 години е приет в кардиологично интензивно отделение с остър трансмурален инфаркт на миокарда, настъпил преди около 5 часа Предписания: анаприлин 20 mg 4 пъти дневно през устата, хепарин венозно 10 000 единици на всеки 4 часа. време, беше възможно да се постигне увеличаване на времето за съсирване на кръвта до 18-23 минути. На 4-ия ден пациентът е диагностициран с туриум микрохема (22 червени кръвни клетки на зрително поле). Каква е вашата тактика?
Отговор: Намалете дозата на хепарина, докато времето за съсирване стане поне 10-12 минути

91. е приет с остър миокарден инфаркт, настъпил преди 5 часа Предписания: анаприлин 20 mg 4 пъти на ден перорално, хепарин интравенозно 10 000 единици на всеки 4 часа В същото време е възможно да се увеличи времето за съсирване на кръвта до 18-23 минути . На следващия ден той беше диагностициран с дясностранна пневмония на долния лоб.Бензилпеницилин натриева сол (1 000 000 единици на всеки 4 часа) беше предписан интравенозно.След 4 часа времето за съсирване на кръвта беше 8 минути. Каква е вашата тактика?
Отговор: Променете начина на приложение на пеницилин

92. Направена е радикална операция за рак на стомаха. На 4-тия ден след операцията при коагулограмата се установява хиперкоагулация и намаляване на фибринолитичната активност на кръвта.. Препоръчително ли е да се предписват антикоагуланти?
Отговор: Показани са антикоагуланти, но е необходимо внимателно наблюдение за предотвратяване на хеморагичен синдром

93. Тя е приета в болницата с оплаквания от силна слабост и задух при ходене. При изследване на кръвта се установява анемия (хемоглобин - 56 g/l), цветен индекс 1,2, при преглед на езика - глосит Пункцията на костен мозък разкрива мегалобластичен тип хемопоеза Концентрацията на желязо в кръвния серум е в рамките на нормата граници Диагноза: В 12 -дефицитна анамия Изберете най-оптималния вариант на лечение.
Отговор: Вит.В12 в доза 500 mcg/ден през ден, фолиева киселина в доза 1,5 mg/ден, железен сулфат (80 mgFe2+) веднъж дневно

94. след хипотермия се появи втрисане, телесна температура се повиши до 38,6 С, кашлица със слузно-гнойни храчки, болка в дясната половина на гръдния кош.Клинично и рентгенологично е поставена диагноза дясностранна долнолобна пневмония.Назначено е лечение на B: цефазолин 0,5 g 2 пъти на ден IM, hemodez 400 ml интравенозно, отхрачваща смес 1 супена лъжица 6 пъти на ден Изберете антиоксидантното лекарство, което е най-добро
ефективно повлияване на процесите на свободнорадикално окисление в белите дробове, което трябва да се добави към терапията
Отговор: Вит.С

95. за остър исхемичен мозъчно-съдов инцидент на 12 часа, получава реополиглюкин 400 ml венозно.
1 рубла на ден Изберете най-ефективното лекарство в тази ситуация, което има антиоксидантни свойства
Отговор: Церебролизин

96. От 5 години страда от деформираща артроза на долните крайници с тежък синовит. Има история на употреба на наркотици
алергии (към бутадион, хепарин, метиндол, пеницилин, теофилин).В болницата са предписани реопирин 5 ml IM веднъж дневно, хидрокортизон хемисукцинат 100 mg в кухината на коленните стави, tavegil 0,001 g 2 пъти на ден. След 3 дни при пациента се появяват сърбящи еритематозни обриви по кожата на торса.Каква е най-вероятната причина за влошаване на състоянието?
Отговор: Лекарствена алергична реакция

97. Диагнозата ревматоиден артрит е потвърдена Какви основни лекарства можете да предпишете за лечение на ревматоиден артрит: а) 4,7-хлорохинолонови лекарства (делагил), б) цитостатици (азатиоприн, циклофосфамид и др.), в) глюкокортикоиди ( преднизолон), г) НСПВС,
д) златни препарати (кризанол), е) салазопиридазин,
ж) антибиотици (тетрациклини), з) D-пенициламин,
и) Имуномодулатори (левамизол)
Отговор: a, b, d, f, h, i

98. На пациент с ревматоиден артрит е предписан метотрексат. Колко време отнема метотрексатът да подейства?
Отговор: След няколко месеца

99. метотрексат е предписан за ревматоиден артрит. Какви мерки ще предприемете, за да наблюдавате безопасността на фармакотерапията с метотрексат при този пациент: а) Пълна седмична кръвна картина
(за предпочитане два пъти седмично), б) Извършване на кръвен тест за определяне на броя на тромбоцитите на всеки 3-4 седмици,
в) Провеждане на общ тест на урината, г) Определяне на съдържанието на пикочна киселина, д) Провеждане на тест за скрита кръв в изпражненията, е) Определяне на съдържанието на трансаминази, общ билирубин на всеки 6-8 седмици
Отговор: a, b, c, d, f

100. за ревматизъм, той приема делагил от дълго време Какви мерки ще предприемете, за да следите безопасността на терапията с делагил при продължително приложение: а) общ кръвен тест, б) общ тест на урината, в) ЕКГ, г) Изследване на очното дъно, д) Изследване на зрителното поле, е) Рентгеново изследване на гръдните органи, ж) Определяне на броя на тромбоцитите, з) Изследване на роговицата
Отговор: a, b, c, d, e, g, h

101. 39 години ревматоиден артрит, предимно ставна форма, 2 степен на активност.Какви варианти на комбинирана терапия е препоръчително да се предпишат за този пациент?
Отговор: Delagil 0,25 g 3 пъти на ден, преднизолон 15 mg / ден, crisanol интрамускулно 1 ml 5% разтвор веднъж седмично

102. Жена на 63 г. страда от захарен диабет и приема глибенкламид.Постъпва в отделение с картина на остра десностранна долнолобна пневмония, потвърдена рентгеново.Назначен е хлорамфеникол, към който пациентката е алергична. реакция.Лекарството е спряно и е избран друг антибиотик цефтриаксон.Въпреки това, по време на прегледа пациентът е имал алергична реакция.-открит е нисък креатининов клирънс (24 ml/min), в резултат на което цефтриаксон Кое лекарство трябва да продължи лечението?
Отговор: Ципрофлоксацин

103. е приета в отделението с болки в дясната млечна жлеза, Т. е повишена до 39,5 С. ​​Заболяла е преди 3 дни, на 10-тия ден след раждането. При постъпване в отделението в горен външен квадрант на дясна млечна жлеза
на жлезата се открива кожна хиперемия и масивен инфилтрат с флуктуация в центъра.Диагноза: остър десностранен мастит.Б-ная е оперирана. Взета е култура на секреция от рана Определете антибиотика на първи избор.
Отговор: Цефазолин

104. е постъпила в отделението с картина на остър десностранен мастит.Заболява преди 3 дни на 10-тия ден след раждането. Б-ная е оперирана.
Предписан е цефазолин.След 2-рата инжекция на лекарството, 20 минути по-късно се появяват понижение на кръвното налягане, замаяност, гадене, повръщане, неволно уриниране, гърчове. Какво усложнение се е развило при пациента?
Отговор: Анафилактична реакция

105. Б-ная на 21 г. постъпва в отделението с картина на остър десен мастит.Заболява преди 3 дни на 10-ия ден след раждането.Б-ная е оперирана.Б-ная има анафилактична реакция на цефазолин, лекарството беше незабавно спряно
staphylococcus, който произвежда пеницилиназа и Haemophilus influenzae , Изберете антибактериално лекарство, като вземете предвид бактериалната микрофлора и фармакокинетичните характеристики
Отговор: Левомицетин

106. страда от хроничен тонзилит и хроничен холецистит.Изследването на гърлото и жлъчната култура разкриват Staphylococcus aureus, който продуцира пеницилиназа.Отбелязана е анамнеза за алергия към оксацилин.Б-ной е предписан гентамицин.Креатининовият клирънс на Б-ной е 50 мл/мин. . Необходимо ли е да се коригира режимът на дозиране на лекарството? Ако да, тогава как?
Отговор: Намалете честотата на приложение и намалете дозата

107. Жена на 50 г. се оплаква от обща слабост, жажда, често уриниране, сърбеж по кожата и външните полови органи.При прегледа е установено затлъстяване (телесно тегло 96 кг при ръст 168 см.) Кръвна захар 9,9 mmol/l, урина 1 %, реакцията към ацетон е отрицателна. Какви хипогликемични лекарства са оптимални в този случай?
Отговор: Бигуаниди

108. Мъж на 48 години е приет с оплаквания от притискаща болка, която се появява при физическо натоварване и се облекчава от нитроглицерин. Преди 3 години прекарах инфаркт на миокарда. Везикуларно дишане в белите дробове. Сърдечните звуци са приглушени, систоличен шум на върха, чести екстрасистоли. Пулс - 92 в минута. Кръвно налягане - 100/60 mm Hg. Изкуство. Черният дроб не е увеличен, няма оток. ЕКГ - синусова тахикардия, цикатрициални промени в миокарда, чести камерни екстрасистоли. Предписани са Obzidan 160 mg / ден, Sustac-Forte 19,2 mg / ден, Panangin, Riboksin.
Какъв страничен ефект е вероятен за пациент с тази комбинация от лекарства?
Отговор: Хипотония, световъртеж.

109. Пациент М., 52 години, е приет с оплаквания от задух, сърцебиене, болка в дясното подребрие, отоци на краката. От 18 години е диспансерно регистриран с диагноза ревматизъм. Бледа кожа, акроцианоза, зачервени бузи. В базалните части на белите дробове има тихи фини хрипове. Границите на относителната тъпота на средата са разширени нагоре и надясно. Сърдечните шумове са приглушени, аритмични, систоличен шум на върха, акцент?? звуци на белодробната артерия. Пулс-96 в минута. Пулс - 140 в минута. Кръвно налягане - 130/85 mm Hg. Изкуство. Коремът е мек, черният дроб излиза на 3-4 см от ръба на ребрената дъга. Подуване на краката. Дневна диуреза -650 мл. ЕКГ: няма P вълна, има вълни F-F, ритъмът е неправилен. След интравенозно приложение на 10 ml 10% разтвор на прокаинамид: синусовият ритъм се възстановява със сърдечна честота 72 на минута, на пациента се предписва прокаинамид 0,5 g перорално 4 пъти на ден, дигоксин 0,25 mg 1 таблетка.
3 пъти на ден, фуроземид 40 mg перорално в продължение на 3 дни. След 5 дни пациентът получава гадене, повръщане, диария и световъртеж. ЕКГ: синусов ритъм, сърдечна честота - 76 в минута, PQ -0,20 s, QRS - 0,1 s. Лекуващият лекар преустанови дигоксин и фуроземид и предписа унитиол и калиеви добавки.
Оценете действията на лекаря.
Отговор: Действията на лекаря са правилни, тъй като не само се надвишава средната дневна доза дигоксин, но има и взаимодействие с прокаинамид поради свързването му с протеина.

110. Пациент Д., 53 години, е диагностициран с коронарна артериална болест, стабилна стенокардия FC III, постинфарктна кардиосклероза, предсърдно мъждене, CNC??B st. Приема строфантин, дигоксин, фуроземид и панангин в средни терапевтични дози. Неочаквано температурата на пациента се повиши до 38,4 ° C, появиха се кашлица, задух и крепитус в белите дробове вдясно. Рентгенова снимка на белите дробове вдясно в долния лоб разкрива зона на инфилтрация. Към лечението са добавени гентамицин, сулфокамфокаин и супрастин.
Какви странични ефекти от лечението е най-вероятно да се появят при пациент с такава комплексна терапия?
Отговор: Когато се комбинира с фуроземид, нефротоксичният ефект на гентамицин е най-вероятен.

111. Пациент на 28 години е приет с оплаквания от сърцебиене, главоболие и втрисане. По време на криза, която се развива 2-4 пъти годишно, кръвното налягане се повишава до 260/110 mm Hg. Чл., Сърдечна честота - 140 в минута, бледа кожа, пареща болка в областта на сърцето, пулсация в главата, понякога повишаване на телесната температура до 38. След пристъпи на полиурия. По време на междупристъпния период кръвното налягане е 120/80 mm Hg. Изкуство. При обективно изследване не се установи органична патология от страна на вътрешните органи. Изследванията на кръвта и урината не показват патология.
Посочете най-ефективното лекарство (първа линия) за облекчаване на криза при пациент:
Отговор: Фентоламин.

112. ревматоиден артрит по време на курс на лечение с метотрексат се появи тежко кървене от носа Какво може да бъде причинено от: а) Увреждане на носните съдове поради подлежащия патологичен процес, б) Повишен PI поради токсичен хепатит, причинен от метотрексат, в) Повишен тромбоцитна агрегация под въздействието на метотрексат,
г) Индуцирано от лекарства намаляване на броя на тромбоцитите, д) Токсичен ефект на метотрексат върху назалните съдове
Отговор: d,d

113. Пациент К., 62 години, има хипертония от 1 стадий. Последното влошаване на състоянието се дължи на психо-емоционален стрес. При преглед: състоянието е относително задоволително, слабо главоболие. BP-170/100 mm Hg. („работно“ кръвно налягане - 120/70 mm Hg), сърдечна честота - 90 в минута. Лекуващият лекар предписва анаприлин 60 mg дневно, верапамил 160 mg дневно.
Какви промени можете да очаквате, когато верапамил се предписва в допълнение към анаприлин?
Отговор: Засилване на отрицателния дромотропен ефект.

114. Пациент С., 56 години, приема нитросорбид (10 mg) 1 х 4 пъти на ден за ангина. Как ще се промени тактиката на антиангинозната терапия, ако пациентът получи мозъчен инсулт по време на лечение с нитрати?
Отговор: Прекратете нитратите и предпишете антиангинозно лекарство от друга група.

115. Пациент на 42 години с хроничен гломерулонефрит и артериална хипертония. При постъпване: артериално налягане 200/120 mm Hg, пулс 75-80 удара в минута, отоци по лицето, кръста и краката. Общият серумен протеин е 3,8 g%, протеинът в урината е 16 g / l. Посочете най-ефективните лекарства за антихипертензивна терапия при този пациент:
Отговор: Еналаприл.

116. Лекарят прилага подкожно 1 ml адреналин на пациент, страдащ от персистиращ рецидивиращ синдром на бронхиална обструкция.
ефектите на адреналина са възможни в тази ситуация: а) Възбуждане на централната нервна система, б) Екстрасистолия, в) Токсично увреждане на черния дроб, г) Тахикардия, д) Блокада на предаването на импулси през проводната система на сърцето.
Отговор: а,б,г

117. 57-годишен мъж получава 40 mg фуроземид IV и 300 mg за слединфарктна ардиосклероза, застойна сърдечна недостатъчност степен 2В.
veroshpiron перорално Каква диуретична терапия ще предпише на пациента в случай на рефрактерност?
Отговор: Фуроземид 80 mg IV и спиронолактон 300 mg перорално

118. страда от неатопична бронхиална астма, съпроводена с профузна бронхорея.Пулс 62 в мин. Кръвно налягане 140/80 mmHg. След предписване на атропин сулфат състоянието на пациента първоначално се подобри, бронхореята рязко намаля, но 10 дни след началото на лечението отново се влоши: температура (37,8 ° С), задух, кашлица с трудно отделими храчки, сърцебиене скорост 90 на минута. Какви са причините за такива промени в състоянието на пациента?
Отговор: Нарушено отделяне на храчки с последваща инфекция

119. Жена на 52 години страда от хипертония?? Изкуство. Приема резерпин по 1 табл. (0,0001) 3 пъти на ден. Кръвното налягане се нормализира след 1 седмица. След 4 седмици редовна употреба се появиха "гладни" болки в епигастричния регион и по време на гастроскопия беше диагностициран ерозивен дуоденит. Как си обяснявате появата му?
Отговор: Повишен тонус на n вагуса на фона на резерпин и повишена стомашна секреция.

120. Пациент на 60 години с коронарна болест на сърцето, стабилна ангина пекторис IV. Предписан е Cordarone 600 mg / ден (като антиангинозно лекарство).
Какви нежелани реакции може да изпита пациентът, когато приема кордарон продължително време?
Отговор: Всички изброени

121. При избора на режим на дозиране на лекарства на базата на Т?
дефинирам:
Отговор: честота на приемане

122. По-точно характеризира скоростта на елиминиране на лекарствата от тялото:
Отговор: общ просвет

123. Връзка на лекарства с плазмени протеини:
Отговор: определя възможността за развитие на странични ефекти при комбиниране на лекарства

124. Количеството бионаличност е важно да се определи:
Отговор: пътища на приложение на лекарства*

125. При продължителна употреба на силни диуретици може да възникне следното:
Отговор: нарушен глюкозен толеранс

126. Замаяност, липса на усещане в крайниците, затруднено сядане и изправяне без визуален контрол и други симптоми на токсични ефекти се появяват при 75% от пациентите, които:
Отговор: получават стрептомицин

127. Предозирането на симпатикомиметици причинява:
Отговор: ритъмни нарушения

128. Нежеланите реакции, свързани с антибиотика моксалактам, включват следното:
Отговор: тромбоцитопения

129. Комбинираната употреба на индометацин и гентамицин най-често причинява:
Отговор: бъбречна дисфункция

130. Използването на клавуланова киселина в комбинация с амоксицилин позволява:
Отговор: разширяване на спектъра на действие на амоксицилин върху щамове бактерии, които произвеждат бета-лактамаза

131. Предвижда се допълнително да се предпише друго антиаритмично лекарство от клас 1 на пациент, получаващ дълго време дифенин.При предписване на кой антиаритмичен препарат ще е необходимо да се увеличи дозата с 20-30% от стандартната?
Отговор: всички лекарства

132. Едновременното перорално приложение на тетрациклин и Ca2+ лекарства ще допринесе за:
Отговор: намалена абсорбция на тетрациклин

133. Едновременното приложение на хлорамфеникол и аценокумарол може да доведе до:
Отговор: за намаляване на антибактериалната активност на хлорамфеникола

134. За сърдечна недостатъчност:
Отговор: допаминът причинява вазоконстрикция на бъбречната кора във високи дози (повече от 10 mcg/kg/min)

135. Артериалната хипертония се характеризира с:
Отговор: повишаване на концентрацията на натрий в съдовата стена

136. Апресин (хидралазин):
Отговор: причинява тахикардия

137. Бета-блокерите причиняват:
Отговор: намаляване на сърдечната честота

138. Следните твърдения за алфа-блокерите са верни:
Отговор: това е вярно

139. Бета-1 - адренергични блокери:
Отговор: селективно действат върху бета1-адренергичните рецептори, лекарствата са безопасни за бронхиална астма

140. Показания за употребата на бета-блокери са:
Отговор: нарушения на сърдечния ритъм

141. Моля, посочете верните твърдения:
Отговор: строфантинът до голяма степен се разрушава в стомашно-чревния тракт и следователно приемането му през устата е нерационално

142. Показания за употреба на SG:
Отговор: CNC при пациенти с исхемична болест на сърцето, постинфарктна кардиосклероза и перманентна форма на предсърдно мъждене

143. Фактор, който повишава риска от развитие на SG интоксикация:
Отговор: хипокалиемия

144. Състояния, които повишават риска от развитие на интоксикация с SG:
Отговор: хипотиреоидизъм

145. За да намалите риска от развитие на толерантност към нитрати:
Отговор: правете паузи между лекарствата

146. Ако се развие толерантност към сустак, той може да бъде заменен с:
Отговор: корватон

147. Главоболието може да бъде причинено от:
Отговор: Отговорите A, B, C са верни

148. Механизмът на действие е подобен на нитроглицерина:
Отговор: Молсидомин

149. Предозирането на кои лекарства може да причини ортостатична хипотония?
Отговор: нитрати

150. Назовете група антиаритмици, които увеличават продължителността на потенциала на действие:
Отговор: блокери на калиеви канали

151. Кое от следните лекарства има най-силно изразен отрицателен инотропен ефект?
Отговор: дизопирамид

152. Кои несърдечни странични ефекти са характерни за повечето лекарства от клас 1C?
Отговор: зрително увреждане

153. Следното заболяване може да се влоши по време на терапия с дизопирамид:
Отговор: доброкачествена хиперплазия на простатата с уринарна дисфункция

154. В какви случаи трябва да се промени дозировката на лидокаин спрямо стандартната?
Отговор: при пациенти с чернодробна недостатъчност

155. Посочете методи за наблюдение на ефективността на диуретиците при едематозен синдром:
Отговор: това е вярно

156. Посочете методи за наблюдение на безопасността на употребата на диуретици за синдром на оток:
Отговор: това е вярно

157. Посочете ефективен и безопасен начин за попълване на запасите от калий в организма:
Отговор: приемане на панангин перорално, 2 таблетки 3 пъти на ден

158. Посочете рисковите фактори за страничните ефекти на бримковите диуретици:
Отговор: дневна диуреза над 3 литра след прилагане на диуретик

159. Посочете началото на действие на спиронолактон:
Отговор: 4-5 дни

160. За алкализиране на урината се използват следните методи, с изключение на:
Отговор: калиев цитрат 3 mg на всеки 6 часа

161. Урината може да бъде кисела, когато се използват следните лекарства, с изключение на:
Отговор: метионин

162. Обърнете внимание на неправилните позиции:
Отговор: без грешни позиции

163. Разпределете посочените лекарства според степента на натрупване:
Отговор: неодикумарин

164. Изберете твърденията, които са напълно верни за лекарството стрептокиназа:
Отговор: това е вярно

165. Изберете фактора, който причинява тромбоза или насърчава образуването на тромби:
Отговор: това е вярно

166. Кое от следните лекарства може да намали ефекта на индиректните антикоагуланти?
Отговор: рифампицин

167. Какви нежелани реакции могат да възникнат при употребата на хепарин?
Отговор: Всички изброени

168. Пациент с бронхиална астма, който дълго време е получавал теофилини с продължително действие, е предписан ципрофлоксация поради развитието на инфекция на пикочните пътища. В този случай е необходимо:
Отговор: намалете дозата на теофилин с 30%

169. Дете, получаващо дълго време карбамазепин поради епилепсия, развива бронхообструктивен синдром с респираторна недостатъчност от 2 етап. Когато се предписва аминофилин на такъв пациент:
Отговор: дозата на аминофилин трябва да се увеличи с 1,5 пъти

170. Когато предписвате теофилин на пушач:
Отговор: дозата трябва да се увеличи

171. Посочете лекарство, което намалява елиминирането на теофилин при едновременно приложение:
Отговор: циметидин

172. Пациент с бронхиална астма, който дълго време приема теотард, разви гадене, повръщане, главоболие и безсъние на фона на грипна инфекция и треска. Менингеалните симптоми са отрицателни. Терапевтична тактика в този случай:
Отговор: спрете теотард или намалете дозата му с 50%

173. Страничните ефекти на теофилин могат да включват следното с изключение на:
Отговор: развитие на едематозен синдром

174. Специфичен страничен ефект, който възниква при употреба на теофилин при деца на възраст 1 година е:
Отговор: мелена

175. Посочете инхалаторния глюкокортикостероид с най-ниска бионаличност:
Отговор: флутиказон пропионат

176. Посочете инхалаторното глюкокортикостероидно лекарство, което има най-нисък афинитет към глюкокортикостероидните рецептори в човешкия бял дроб:
Отговор: флутиказон пропионат

177. Посочете лекарството с най-висока степен на безопасност (според индекса на безопасност:)
Отговор: преднизолон

178. Кое глюкокортикостероидно лекарство най-много допринася за развитието на миопатия?
Отговор: триамцинолон

179. Забавянето на отделянето на натрий и вода от тялото, увеличаването на отделянето на калий (минералокортикоиден ефект) е по-характерно за:
Отговор: хидрокортизон

180. Минералокортикоидната активност липсва при:
Отговор: дексаметазон

181. Изберете верният отговор. Глюкокортикоиди:
Отговор: са контраинсуларни хормони

182. При провеждане на импулсна терапия е по-предпочитано:
Отговор: метилпреднизолон

183. При дългосрочно предписване е за предпочитане да се използват:
Отговор: преднизолон

184. Кой H1-хистамин рецепторен блокер е противопоказан при анафилактичен шок?
Отговор: дифенхидрамин (дифенхидрамин)

185. Изберете оптималния H1-хистаминов рецепторен блокер за лечение на алергичен ринит:
Отговор: азеластин (алергодил)

186. Посочете лекарство от групата на стабилизаторите на мембраната на мастоцитите в дозирана форма под формата на прах за инхалация:
Отговор: кромоглицинова киселина (бикромат)

187. Имуностимулатор от микробен произход включва:
Отговор: рибомунил

188. Основните показания за предписване на Ribomunil са:
Отговор: предотвратяване на рецидивиращи инфекции на горните дихателни пътища

189. Следните антибактериални лекарства проникват добре през кръвно-мозъчната бариера:
Отговор: III поколение цефалоспорини

190. Новото поколение макролидни антибиотици има следните предимства с изключение на:
Отговор: път на бъбречна екскреция

191. Флуорохинолоните се различават от хинолоните по следните начини, с изключение на:
Отговор: бактериостатичен ефект

192. Проверете кои твърдения относно цефалоспорините са верни:
Отговор: това е вярно

193. Страничните ефекти от приема на антибиотици включват:
Отговор: това е вярно

194. Посочете избраното лекарство за инфекция на пикочните пътища, причинена от Pseudomonas aeruginosa:
Отговор: цефтазидим

195. Какви лекарства са показани за лечение на хламидийни инфекции на пикочно-половия тракт:
Отговор: ровамицин

196. Посочете лекарството с най-неблагоприятни фармакокинетични характеристики:
Отговор: кетоконазол

197. Посочете антимикотично лекарство, което не се метаболизира в черния дроб:
Отговор: флуконазол

198. Посочете антимикотично лекарство (от групата на алиламините), използвано предимно за лечение на дерматомикоза:
Отговор: тербинафин

199. Посочете клиничното състояние, което е индикация за монотерапия с НСПВС:
Отговор: извънставни ревматични заболявания (миозит, тендовагинит, синовит)

200. Ацетилсалициловата киселина се характеризира с:
Отговор: когато се приема през устата, той се абсорбира главно от горната част на тънките черва

201. В сравнение с индометацин, ацетилсалициловата киселина има по-изразени:
Отговор: антитромбоцитен ефект върху тромбоцитите

202. Скоростта на екскреция на ацетилсалициловата киселина и нейните метаболити се влияе от:
Отговор: ниво на pH на урината

203. Стомашно-чревните усложнения при употребата на ацетилсалицилова киселина са свързани с:
Отговор: Всички изброени

204. Фенилбутазонът се характеризира с:
Отговор: това е вярно

205. Когато индометацин взаимодейства с други лекарства:
Отговор: диуретичната активност на фуроземид намалява

206. Какви нежелани реакции на НСПВС се коригират от комплексното лекарство arthrotek (диклофенак натрий + мизопростол)
Отговор: НСПВС гастропатия

207. Какви характеристики на парацетамола поставят това лекарство на първо място сред аналгетиците и антипиретиците?
Отговор: по-ранно начало на аналгетичен и антипиретичен ефект

208. Изберете лекарство, което селективно инхибира циклооксигеназа2:
Отговор: мелоксикам

209. Най-добър аналгетичен ефект на фентанил се наблюдава в комбинация с:
Отговор: дроперидол

210. Назовете дългодействащо противовъзпалително средство:
Отговор: пироксикам

211. Б, 52 г., развива картина на хипертонична криза тип 2 с сърдечна честота 62 в минута, кръвно налягане 200/140 mmHg, има голям брой влажни, фино мехурчести хрипове в белите дробове. лекарството трябва да се използва за облекчаване на кризата:
Отговор: Фуроземид

212. страда от захарен диабет тип 1 в продължение на 6 години, получава инсулин от 54 единици / ден, което поддържа гликемичното ниво в рамките на 7,0 mmol / l. Наскоро, поради повишаване на кръвното налягане до 16090 mmHg. Лекуващият лекар предписва хипотиазид в дневна доза от 75 mg в комбинация с еналаприл в доза от 5 mg.След 10 дни нивото на кръвната захар на пациента е 10,5 mmol и има влошаване на здравето. Коя е водещата причина за промени в нивата на кръвната захар?
Отговор: Комбинация от еналаприл с хипотиазид

213. развила се конвулсивна форма на хипертонична криза, състоянието е тежко, кръвното налягане е 200-120 mmHg, сърдечната честота е 120 в минута. С какво лекарство трябва да започнете терапията?
Отговор: Диазепам

214. След прилагане на антибиотика цефтриаксон в продължение на 10 дни се разви картина на псевдомембранозен колит. Каква е първата стъпка от алгоритъма за медицинско обслужване?
Отговор: спиране на цефтриаксон, приложение на ванкомицин или метронидазол

215. Кларитромицинът е предписан като част от терапията за обостряне на стомашна язва. Какви са основните отличителни черти на лекарството от еритромицин?
Отговор: това е вярно

216. След операция на коремната кухина на 4-ия ден се разви левостранна долнолобна пневмония. Резултатите от бързия анализ показват наличие на MRSA, резистентни към пеницилин и аминогликозиди щамове ентерококи. Лекарства по избор:
Отговор: Ванкомицин

217. е в интензивно отделение за псевдомонас инфекция. Изберете лекарства от първа линия за лечение?
Отговор: Цефтазидим + аминогликозиди

218. На 40-годишен пациент без съпътстващи заболявания е предписан перорално спирамицин в доза от 3 милиона IU 2 пъти на ден за придобита в обществото пневмония на амбулаторна база; на 2-ия ден от лечението, интензивна гастралгия, гадене и еднократно повръщане бяха отбелязани. Изберете алтернативно лекарство.
Отговор: доксициклин

219. с хроничен обструктивен бронхит се открива умерена пневмония; амбулаторно се предписва амоксиклав перорално в доза от 625 mg 3 r. На 2-ия ден пациентът развива уртикария и бронхоспазъм. Назовете алтернативно лекарство за лечение на пневмония.
Отговор: моксифлоксацин перорално

220. 44-годишен ХИВ-инфектиран мъж е диагностициран с пневмоцистна пневмония. Назовете лекарството за лечение?
Отговор: ко-тримоксазол IV 20 mgkgs 4 пъти за 21 дни

221. Б-ной има 28л. отбелязват се ежедневни симптоми на бронхиална астма, чести обостряния, чести нощни симптоми, диагностицирана е тежка персистираща бронхиална астма. Назовете основните лекарства за терапия.
Отговор: инхалаторни глюкокортикоиди (повече от 1000 mcg беклометазон дипропионат) + дългодействащи инхалаторни бета-2 агонисти

222. Бременна жена (гестационна възраст 6-7 седмици) се консултира с лекар с клинични признаци на остра пневмония. Какви групи антибактериални лекарства са разрешени за употреба при бременни жени?
Отговор: цефалоспорини

223. Мъж на 57 години е на монотерапия с АСЕ инхибитор еналаприл за умерена артериална хипертония. След 2 години прием на лекарството пациентът показва недостатъчен ефект. Кой е най-подходящият вариант за оптимизиране на терапията?
Отговор: добавяне на диуретик (хипотиазид или индапамид) към лекарството

224. получава антибактериално лекарство за инфекциозен процес. При интравенозна инфузия на лекарството се наблюдава реакция под формата на изразено зачервяване на кожата на горната половина на тялото, лицето и шията; симптомите намаляват значително, когато скоростта на инфузия намалява. Към какво лекарство възниква тази реакция?
Отговор: Ванкомицин

225. При бременна жена се наблюдава активиране на ревматичния процес. Кое лекарство от групата на антикоагулантите може да се предпише на бременна жена?
Отговор: Хепарин

226. приет с искане за повишаване на артериалното налягане до 15090 mmHg. на фона на психо-емоционален стрес, сърцебиене, тревожност, нарушение на съня. Преди година му откриха диабет тип 2 и приема манинил. Назовете избраното лекарство за лечение на хипертония.
Отговор: Атенолол

227. Поради симптомите на ангина пекторис и ритъмни нарушения са предписани лекарства: анаприлин 200 mg и верапамил 240 mg за дълго време. Какви са възможните странични ефекти?
Отговор: Развитие на a-v блокади, брадикардия

228. Жена на 34 години приема естроген-съдържащи контрацептиви. Лекуващият лекар предписва доксициклин в доза от 200 mg за 2 седмици. Какви вероятни взаимодействия се очакват?
Отговор: Ефектът от контрацепцията е намален

229. При прилагане на кетамин с профилактична цел, анестезиологът предписва на пациента диазепам. Какво състояние се предотвратява по този начин?
Отговор: Халюцинации след анестезия

230. Мъж на 46 години е приет с картина на остър деструктивен апендицит. Лекарство на избор за антибиотична профилактика?
Отговор: Цефазолин

231. Пациент се обърна към лекар с оплаквания от кашлица, температура до 39 С и болки в гърдите. Диагностицирана е дясна бронхопневмония. Предписано е лекарство за 3 дни, което има постантибиотичен ефект. Назовете избраното лекарство.
Отговор: Азитромицин

232. при остър десен пиелонефрит се предписва цефазолин 2 g за 10 дни. Посочете най-честата грешка при избора на този антибиотик
Отговор: Недостатъчно висока активност срещу грам-отрицателна флора

233. едновременно получава флуорохинолонов антибиотик в продължение на 14 дни - офлоксацин за инфекция на пикочните пътища и диклофенак натрий за ставен синдром. Какви вероятни взаимодействия се очакват?
Отговор: повишен риск от възбуждане на централната нервна система и развитие на припадъци

234. след хипотермия се появи втрисане, повишаване на телесната температура до 38,6 С, кашлица със слузно-гнойни храчки и болка в лявата половина на гръдния кош.Клинично и рентгенологично е установена диагноза левостранна пневмония на долния лоб.Назначено е лечение за B: цефазолин 1 g 2 пъти на ден IM, hemodez 400 ml IV капково, отхрачваща смес 1 супена лъжица 6 рубли на ден. На 3-ия ден се наблюдава реакция под формата на уртикария и сърбеж по кожата. Изберете антибактериално лекарство, което да замените?
Отговор: спирамицин

235. По време на анестезията анестезиологът предписва антибактериално лекарство с цел антибиотична профилактика. Пациентът разви респираторен арест. Какво лекарство е използвано?
Отговор: Гентамицин

236. От 10 години страда от деформираща артроза на долните крайници с тежък синовит. Има анамнеза за лекарствена алергия към бутадион. В отделението на пациента предписаха реопирин 5 ml IM веднъж дневно.Ден по-късно пациентът разви сърбящи еритематозни обриви по кожата на торса.Каква е най-вероятната причина за влошаване на състоянието?
Отговор: Лекарствена алергична реакция

237. При сърдечни аритмии беше предписан прокаинамид, а при сезонен алергичен ринит - цетиризин. Какви вероятни взаимодействия се очакват?
Отговор: тежки форми на аритмия (като пирует)

238. На пациент със СЛЕ е предписан метотрексат. Колко време е необходимо, за да се появи стабилен терапевтичен ефект?
Отговор: Бирнеш Айдън Кеин

239. с хипертонична криза е предписано лекарството фозиноприл. Въпреки приемането на адекватна доза от лекарството, в следващите минути и часове не се наблюдава понижение на нивата на кръвното налягане. Дай причина.
Отговор: По време на кризи не се използват депо лекарства

240. За целите на планираното лечение на хипертония дълго време се предписва апресин. След един месец употреба на лекарството пациентът започва да изпитва сърцебиене, болка в стенокардия и намаляване на ефекта от лечението. Посочете основната причина за развитите явления
Отговор: Апресин не се предписва за рутинно лечение на хипертония.

241. 42-годишен мъж е настанен в реанимация с тежка хипертонична криза. Натриевият нитропрусид се прилага интравенозно в продължение на 5 дни. На 6-ия ден пациентът развива картина на интоксикация под формата на неудържимо повръщане, намалена функция на сърдечно-съдовата, дихателната и отделителната система. Посочете основната причина за развилото се състояние.
Отговор: Предозиране на лекарството (натрупване на тиоцианати в кръвта)

242. Жена на 54 г. страда от захарен диабет тип 1 и приема дългодействащ инсулин.Постъпва в отделението с картина на остра дясно-долнолобна пневмония, потвърдена рентгеново.Назначен е цефтриаксон, на който пациентът е имал алергична реакция.Лекарството е спряно и е избран друг антибиотик - ципрофлоксацин в комбинация с амикацин.При преглед обаче се установява ниско ниво на креатининов клирънс (30 ml/min), в резултат на кой амикацин е спрян Кое лекарство трябва да продължи лечението?
Отговор: спирамицин

243. е приета в отделението с болки в долната част на корема, Т. се повишава до 39,5 С. ​​Заболява преди 2 дни, на 6-тия ден след раждането. При гинекологичен преглед се установява картина на остър следродилен ендометрит. Резултати от култура: Staphylococcus aureus, който произвежда пеницилиназа, Proteus. Определете първия избор на антибиотик
Отговор: Цефепим

244. 25 г. е постъпила в отделението с картина на остър десен пиелонефрит, заболяла преди 3 дни след хипотермия. Предписан е цефазолин.След 2-рата инжекция на лекарството, 10 минути по-късно се появяват понижение на кръвното налягане, замаяност, гадене, повръщане, неволно уриниране, гърчове. Какво усложнение се е развило при пациента?
Отговор: Анафилактична реакция

245. Б-ная на 28 г. постъпва в отделението с картина на остър десен мастит.Заболява на 12-тия ден след раждането.Б-ная е оперирана.Б-ная има анафилактична реакция към цефазолин, лекарството е незабавно преустановено Staphylococcus е изолиран при култивиране на секрета от раната , образувайки пеницилиназа и кандида Изберете антибактериално лекарство, като вземете предвид бактериалната микрофлора и фармакокинетичните характеристики
Отговор: Оксацилин + флуконазол

246. страда от хроничен холецистит.При изследване на жлъчна култура са открити Staphylococcus aureus и Escherichia coli. Имаше анамнеза за алергия към оксацилин. Лекарства по избор.
Отговор: Цефтриаксон

247. Жена на 58 години се оплаква от обща слабост, жажда, често уриниране, сърбеж по кожата и външните полови органи.При преглед: телесно тегло 56 кг при ръст 168 см. Глюкоза в кръвта 12,3 mmol/l, в урина 1,5%, реакцията към ацетон е отрицателна. Какви хипогликемични лекарства са оптимални в този случай?
Отговор: Сулфонилуреи

248. 53 г., постъпва с оплаквания от сърцебиене, нередности, понякога задух. Тези явления станаха тревожни след инфаркт на миокарда преди 2 години. Приемът на прокаинамид в продължение на 3 месеца донесе значително облекчение. Напоследък обаче здравето ми се влоши. Допълнителни тактики за лечение на пациента.
Отговор: пълен преглед и избор на лекарство

249. Мъж на 33 години получава комбинация от лекарства за следоперативен гноен перитонит: цефтриаксон + амикацин + метронидазол. Има анамнеза за холелитиаза. Посочете лекарство, което не трябва да се предписва на пациент.
Отговор: Цефтриаксон

250. На 45-годишен мъж е предписан флуконазол интравенозно за 3 дни за кандидозна пневмония, след това перорално. На 4-ия ден от лечението се наблюдава реакция под формата на силно главоболие и гадене. Беше решено да се замени лекарството с кетоконазол. Оценете адекватността на тактиката.
Отговор: Кетоконазол не е адекватен заместител поради неблагоприятни фармакокинетични характеристики

251. 42 г., постъпва с оплаквания от силно сърцебиене, повишено кръвно налягане до 240-140 mmHg. Изследването разкрива значително повишаване на нивото на катехоламините в кръвта. Назовете лекарството по избор за спиране на кризата.
Отговор: фентоламин

252. 50 години, постъпва с картина на остър левостранен пиелонефрит. Предписани са цефазолин + гентамицин в умерени терапевтични дози. По време на прегледа креатининовият клирънс на пациента е 50 mlmin. Какви са вероятните последици от терапията?
Отговор: Риск от нефротоксичност

253. На 48-годишен пациент с пароксизмална супра- и вентрикуларна тахикардия е предписан IV кордарон на първия ден, след това перорално. При прегледа се установи дисфункция на щитовидната жлеза, блокада 2-3 степен. Допълнителни тактики.
Отговор: Спиране на лекарството, прилагане на новокаинамид

254. От 50 години той е в кардиологичното интензивно отделение за остър инфаркт на миокарда и получава комплексна терапия. Какви са основните параметри за наблюдение при предписване на директни антикоагуланти?
Отговор: APTT, време на съсирване на кръвта, урина върху червени кръвни клетки

255. В отделението има пациент след операция на апендицит. Лекуващият лекар предписва гентамицин 80 mg 3 пъти мускулно. Пациентът страда от остър гломерулонефрит преди 2 години, в момента креатининовият клирънс е 50 mlmin. Каква корекция е необходима?
Отговор: замяна с лекарство, което няма нефротоксичност

256. 56 години, през последната година приема дигоксин 0,25 g. В момента се наблюдава повишаване на кръвното налягане до 180110 mmHg. Лекуващият лекар предписва лизиноприл в доза от 10 mg. След 3 месеца пациентът е диагностициран с дигиталисова интоксикация. Допълнителни тактики.
Отговор: предписване на друг вид антихипертензивно лекарство

257. 53 години с диагноза кандидоза и аспергилусен менингит. Лекарства по избор.
Отговор: Амфотерицин Б

258. Мъж на 58 години, приемащ продължително време глюкокортикоиди, е включен в лечебния комплекс със синтетичен широкоспектърен антибиотик поради инфекция на репродуктивния тракт. На 14-ия ден от комбинираната употреба се наблюдава тежко усложнение под формата на разкъсване на ахилесовото сухожилие. Посочете антибиотика, който в комбинация с GCS е причинил това усложнение.
Отговор: Левофлоксацин

259. Б., 43 г., постъпва в отделението с картина на остра хламидийна пневмония. Назовете лекарствата по избор.
Отговор: Ровамицин

260. Пациент на 24 години е диагностициран с неусложнена форма на гонорея. Назовете избраното лекарство.
Отговор: Цефтриаксон

261. Изберете правилното твърдение: а) бионаличността е количеството лекарства, влизащи в системното кръвообращение, изразено като процент от приетата доза, б) бионаличността се определя от степента на адсорбция на лекарството в стомашно-чревния тракт и израза
ефективността на първото преминаване през черния дроб в) бионаличността се определя по формулата: F = AUC (им или перорално)/AUC (iv) г) бионаличността на лекарството, когато се прилага интрамускулно, се определя от степента на неговата абсорбция и биотрансформация в организма.
Отговор: a B C

262. страда от неатопична бронхиална астма, придружена от профузна бронхорея Пулс 62 в минута Кръвно налягане 140/80 mmHg Кои лекарства са за предпочитане?
Отговор: Атровент

263. Механизми на абсорбция на лекарството в червата:
а) пасивна дифузия, б) филтрация, в) активен транспорт, г) улеснен транспорт, д) пиноцитоза:
Отговор: а, г

264. беше приет с киселини, болки в епигастралната област на гладно, облекчени от приема на натриев бикарбонат FEGDS разкри язва (0,5 см в диаметър) в ампула от 12 бр рН-метрия на стомашен сок: образуване на киселина
заядлива функция със средна интензивност с ниски алкални резерви, холинергичен тип рецепция. Диагноза: пептична язва 12 p.k в остър стадий. Изберете най-ефективното и безопасно лекарство и определете неговия режим на дозиране:
Отговор: Пирензепин преди хранене, 0,05 g 3 пъти на ден в продължение на 2 дни, след това 0,05 g 2 пъти на ден

265. Установена е дискинезия на жлъчния мехур от хипертоничен тип. Изберете най-добрия вариант за лечение.
Отговор: Но-шпа по 1-2 таблетки 3 пъти на ден, отвара от безсмъртниче по 1/2 чаша 30 минути преди хранене.

266. страда от хроничен холецистопанкреатит в продължение на 5 години.През последната седмица след прекъсване на диетата той отбеляза повишена болка в десния квадрант, гадене, горчивина в устата.Изберете най-ефективните холеретични средства, които едновременно имат антимикробна активност:
а) Алохол, б) Холензим, в) Никодин, г) Отвара от вратига,
д) Ксилитол
Отговор: a,c

267. Взех 20 таблетки феназепам за суицидни цели.2 часа след приема на лекарството ме закараха в болницата. Б е в съзнание, но рязко потиснат Извършена е стомашна промивка Изберете най-оптималните лаксативи: а) глауберова сол, б) магнезиев сулфат, в) екстракт от кора на зърнастец, г) бисакодил,
д) рициново масло, е) морски водорасли, ж) вазелиново масло
Отговор: а,б,г

268. Мъж на 46 години е приет в кардиологично интензивно отделение с остър трансмурален инфаркт на миокарда, настъпил преди около 5 часа Предписания: анаприлин 20 mg 4 пъти дневно през устата, хепарин венозно 10 000 единици на всеки 4 часа. време, беше възможно да се постигне увеличаване на времето за съсирване на кръвта до 18-23 минути. На 4-ия ден пациентът е диагностициран с микрохематурия (22 червени кръвни клетки в зрителното поле). Каква е вашата тактика?
Отговор: Намалете дозата на хепарина, докато времето за съсирване стане поне 10-12 минути

КАТЕГОРИИ

ПОПУЛЯРНИ СТАТИИ

2023 “kingad.ru” - ултразвуково изследване на човешки органи